Important MCQS for theory exam dermatology

 Regarding erythrasma all true EXCEPT :

A Presents as well-demarcated red-brown macules/patches 
B Treatment with topical antibiotic or antifungal
C Superficial infection due to C. minutissimum
D Coral-red fluorescence in wood’s lamp(coproporphyrin I)
E Found in intertriginous areas

D Coral-red fluorescence in wood’s lamp(coproporphyrin I).Erythrasma is characterized by sharply delineated, dry, brown,slightly scaling patches occurring in the intertriginous areas,caused by the diphtheroid Corynebacterium minutissimum.Wood’s lamp show bright coral-red fluorescence due to porphyrin production ( coproporphyrin III) . solution tolnaftate applied twice a day for 2–3 weeks, and topical miconazole are equally effective.

UVA II encompasses which wavelengths ? 
A. 290-320 nm 
B. 340-400 nm 
C. 400-450 nm 
D. 320-340 nm 
E. 320-400 nm

UVA can be divided into UVA II (320-340 nm) and UVA I (340-400 nm).

All of the following perforating disorders are associated with chronic renal failure EXCEPT:
A. Reactive perforating collagenosis
B. Perforating folliculitis
C. Elastosis perforans serpiginosa
D. Kyrle disease
E. Calciphylaxis

Elastosis perforans serpiginosa (EPS) has no known association with chronic renal failure. EPS can be associated with inherited fibrous tissue abnormalities, D-penicillamine, or idiopathic etiologies. A, B, and D can be associated with chronic renal failure. Answer choice E is not a perforating disorder but is common in patients with end-stage renal disease.

You prescribe oral erythromycin to a 30 year-old woman. Co-administration of which of the following medications could lead to potential adverse outcomes? 
A Oral contraceptives
B Warfarin
C Carbamazepine
D Methylprednisolone
E Warfarin, carbamazepine, or methylprednisolone

E. Erythromycin inhibits the hepatic cytochrome P450 system and can increase serum levels and potential toxicities of carbamazepine, theophylline, warfarin, digoxin, and methylprednisolone.

Acute hemorrhagic edema of childhood is distinguished from Henoch-Schonlein Purpura based on: 
A The presence of pupura on the upper trunk 
B The lack of an antecedent infection 
C The involvement of the synovia 
D The neurologic complications 
E The lack of systemic features

E Acute hemorrhagic edema of childhood affects children and infants < 2 years of age. It presents with painful, edematous petechiae and ecchymoses on the head and distal extremities. Facial edema may be the initial sign. Triggering factors include infection, drugs, and immunization. It lacks the systemic features of HSP, and resolves in 1-3 weeks without sequelae. HSP occurs mostly in children. There is an antecedent URI in 75% of cases. HSP involves the skin, synovia, GI tract, and kidneys. Long-term morbidity results from renal disease, which is predicted by the spread of purpura to the upper trunk.

A patient had antibodies to desmoglein 3, but no antibodies to desmoglein 1 or desmplakin. The likely diagnosis is: 
A. erythema multiforme 
B. bullous pemphigoid 
C. pemphigus vulgaris 
D. pemphigus foliaceous 
E. dermatitis herpetiformis

The answer is pemphigus vulgaris, which typically has autoantibodies to desmoglein 3. While mixed forms of pemphigus can occur, the histology in this case confirmed pemphigus vulgaris.

A patient with Carney complex has distal arthrogryposis and is autosomal dominant. The heart findings are: 
A. Atrial myxomas 
B. Ventricular hypertrophy 
C. Dilated blood vessels 
D. Third degree block 
E. Second degree block

A patient with Carney complex has atrial myxomas. Early diagnosis can save lives. It is a mutation in the PRKAR1A gene

A 17-year-old female originally presents due to a swelling around her earlobe. She had her ears pierced around three months ago and has noticed the gradual development of an erythematous swelling since. On examination a keloid scar is seen. What is the most appropriate management?
A. Intralesional diclofenac
B. Advise no treatment is available
C. Intralesional triamcinolone
D. Advise will spontaneously regress within 4-6 months
E. Intralesional sclerotherapy

Keloid scars are tumour-like lesions that arise from the connective tissue of a scar and extend beyond the dimensions of the original wound. early keloids may be treated with intra-lesional steroids e.g. triamcinolone. excision is sometimes required. 

A patient is diagnosed with netherton's syndrome and has invagination of the distal hair shaft into the cup formed by the proximal hair shaft also known as:
A. Trichorrhexis invaginata
B. Trichorrhexis nodosum
C. Trichoschisis
D. Trichothiodystrophy
E. Pili torti

Patients with Netherton's syndrome have trichorrhexis invaginata. This syndrome also includes
ichthyosis linearis circumflexa, atopy, trichorrhexis nodosa. They have a mutation in the
SPINK5 gene with mutation in the serine protease inhibitir

What is the most abundant collagen found on fetal skin?
A. Type I Collagen 
B. Type II Collagen 
C. Type IV Collagen 
D. Type VII Collagen 
E. Type III Collagen

 

Type III collagen is found in the fetal skin. It is also present in the gastrointestinal tract, blood vessels, and the basement membrane. A defect in this collagen results in the Ehlers-Danlos, vascular type.

 

1A college student does not go to her scheduled class. She has fever, sore throat, malaise, fatigue and a bump in the right side of the neck. Her abdominal exam showed a increased liver size. The microscopic exam of the epithelial cells revealed a giant nuclei surrounded by clear zones. The cause of the infection is:
A. Herpes virus type 1
B. HIV
C.Cytomegalovirus
D. Herpes virus type 2
E. Papilloma virus

 

Cytomegalovirus produces the owl’s eye nuclei (giant nuclei surrounded by clear zones).

 

All of the following ichthyoses have an associated ocular finding EXCEPT:
A. Conradi-Hunerman syndrome
B. X-linked ichthyosis
C. Netherton syndrome
D. Sjogren-Larson syndrome
E. Refsum syndrome

 Netherton syndrome patients do not have ocular manifestations of disease. X-linked ichthyosis is associated with comma-shaped corneal opacities. Sjogren-Larson syndrome patients show “glistening dots” of the retina by 1 year of age. Refsum syndrome is associated with “salt and pepper” retinitis pigmentosa, night blindness, and cataracts. Conradi-Hunerman syndrome is associated with focal cataracts.

 

What is the antigen associated with dermatitis herpetiformis?
A. Tissue transglutaminase
B. Desmocollin 
C. Epilegrin 
D. Desmoglein 3 
E. Periplakin

MCQ Dermatology Dermatitis herpetiformis is an autoimmune blistering disease associated with intense pruritus. Classically, it involves the extensor aspects of the body. Antibodies to tissue transglutaminase (anti-endomesial, anti-gliadin, anti-reticulin) can be found. It is associated with celiac sprue. 50% have thyroid disease.

 



 

21. Which of the following is not a major criterion for Kawasaki’s disease: 
A. Palmoplantar erythema > desquamation 
B. Cardiac aneurysm
C. Fever >5 days 
D. Strawberry tongue/ red lips 
E. Cervical adenopathy

MCQ Dermatology Cardiac aneurysm is a serious complication of Kawasaki’s disease. However, as the cardiovascular manifestations generally present 1 –5 months after presentation, they are not criteria for diagnosis.

 

 

22. What is the most common ocular finding in Sarcoidosis disease? 
A Blepharitis
B Acute anterior uveitis
C Posterior uveitis
D Keratoconus
E Cataract

 

Sarcoidosis is a systemic disease characterized by the formation of non-caseating granulomas. Organs involved include the skin, eyes, lungs, liver and spleen. The most common ocular finding is acute anterior uveitis. Other findings may include posterior uveitis, blurred vision, and excessive lacrimation. 


An overweight, post-menopausal woman presents with intense pain in the fatty deposits on her knees, thighs and hips. She also describes swelling of her hands and feet, associated fatigue, and history of depression. On clinical exam the pain in the adipose tissue appears out of proportion to the clinical findings. The diagnosis is: 
A Angiolipomas
B Adiposis dolorosa
C Congenital lipodystrophy
D Lipodermatosclerosis
E Erythema nodosum


Adiposis dolorosa (Dercum's disease) is a disorder that usually occurs in obese women 40 to 60 years of age, and it is characterized by pain in adipose tissue that appears to be out of proportion to the physical findings. The pain can be localized to multiple, painful lipomas on lower extremities and knees. This condition is accompanied by swelling of different areas of the body, such as hand and feet, which may be transient. Patients typically have subjective fatigue or confusion, and may have history of depression or emotional instability. The cause of adiposis dolorosa is not known. The cause of the pain is speculated to be due to the pressure on nerves by the adipose tissue deposits. Therapeutic treatments are not very effective, and can involved procedures such as liposuction, systemic corticosteroids, pregabalin, lidocaine, and psychiatric care.

 

 

24. What is the most common form of psoriatic arthritis? 
A Arthritis mutilans 
B Axial 
C Symmetric polyarthritis 
D Asymmetric oligoarthritis 
E Symmetric oligoarthritis


Approximately 70 % of psoriatic arthritis is asymmetric oligoarthritis arthritis mutilans occurs in about 5% of patients with psoriatic arthritis.



Renewal of the epidermis takes approximately:
A. 13–14 days
B. 1–14 days
C. 14–26 days
D. 26–28 days
E. 28–32 days


D. It takes 13–14 days for maturation of keratinocytes from the basal layer to the corneum and another 13–14 days for shedding.

 

 

Which of the following is the shortest visible wavelength?
A. X-ray
B. Infrared
C. Gamma rays
D. Radio waves
E. ultraviolet


C From shortest to longest wavelength: gamma rays, x-ray, ultraviolet,visible, infrared, microwave, radio wave.

 

43. Which of the following mycobacterial organisms is classified as a photochromogen?
A. M. tuberculosis 
B. M. grodonae 
C. M. Kansasii
D. M. ulcerans 
E. M. fortuitum 


M. Kansasii, M. marinum, and M. simiae are photochromogens. They form pigment when exposed to light.

Which biologic agent is administered intramuscularly?
A. Infliximab 
B. Efalizumab 
C. Etanercept 
D. Alefacept 
E. None of the above 

 

Alefacept (anti-CD2) biological therapy selectively targets effector memory T cells (Tem) in psoriasis vulgaris, a model Type 1 autoimmune disease. Alefacept is given intramuscularly.

 

Upon histologic examination of a vascular neoplasm, tufts of capillaries in a cannonball pattern are seen throughout the dermis. What is the most likely diagnosis?
A. Kaposi sarcoma
B. Pyogenic granuloma
C. Tufted angioma
D. Infantile hemangioma
E. Kimura’s disease 

 

Histologic examination of tufted angioma shows tufts of capillaries throughout dermis in a “cannonball” pattern.

 

A syncytial cell found within granuloma is a
A M1 macrophage
B M2 macrophage
C Kupffer cell
D Foam cell
E Giant cell 


E. Syncytial cells are commonly found in granulomas as a result of macrophage fusion to form giant cells.

 



47. The emergency department rings you for advice about a patient who claims to have an allergy to corticosteroids. The patient has been admitted with an exacerbation of inflammatory bowel disease and the team are keen to start systemic steroids. You have the patient’s recent patch testing results to hand which showed a 3+ reaction to tixocortol-21-pivalate, the patient is awaiting further patch testing to the steroid series. Which of the following steroids is likely to be the safest:
A Hydrocortisone
B Methylprednisolone
C Dexamethasone
D Prednisolone
E Diflucortolone 


C Tixocortol-21-pivalate is commonly used as a steroid marker of allergic contact dermatitis, this alone picks up over 90% of steroid reactions. A positive reaction often indicates allergy to class A steroids – hydrocortisone, prednisolone, diflucortolone, methylprednisolone and prednicarbate.


The primary histologic feature of eczema is
A Spongiosis
B Acanthosis
C Hyperkeratosis
D Vasodilation
E Lymphocytic infiltration 

Eczema is a common skin condition with multiple clinical patterns, characterised histologically by a spongiotic tissue reaction pattern


A patient with multiple facial trichilemmomas is at risk of which of the following cancers?
A. Basal cell carcinoma
B. Cylindroma
C. Breast carcinoma 
D. Oral squamous cell carcinoma
E. Acute leukemia 

C. Cowden syndrome (multiple hamartoma syndrome) is an autosomal dominant disorder caused by
mutations in PTEN, a phosphatase that dephosphorylates tyrosine, serine, and threonine.
Clinically, there are numerous facial trichilemmomas, oral papillomas, acral keratotic papules,
sclerotic fibromas, breast fibroadenomas and adenocarcinomas, thyroid adenomas and
adenocarcinomas, and hamartomatous polyps of the gastrointestinal tract 

When does inosculation occur after skin graft placement?
A. First 48 hours
B. Days 2 to 3
C. Days 4 to 7
D. Day 4 to week 4
E. After 2 to 3 months 

B. Day 2 to 3. A skin graft is any skin that is detached completely from its blood supply, removed from its donor site, and transplanted to a recipient site for wound closure in the same individual. Graft survival depends on the establishment of new vasculature between the wound recipient site and the donor graft through the following phases: imbibition, inosculation, capillary ingrowth and neovascularization, keratinocyte activation, and finally,sensory innervation.

 

 

A 4-year-old girl,develop multiple warty papules on the dorsum of the hands, face and eyelid margins. you noted hoarse voice..histological findings on biopsy show periodic acid-Schiff- positive deposition of amorphous hyaline material in the papillary dermis. the most likely diagnosis:
A Hydroa vacciniforme
B Scleromyxedema
C Creutzfeldt–Jakob disease
D Lipoid proteinosis
E Leprosy 


Lipoid proteinosis (Urbach-Wiethe disease) is a rare autosomal recessive disorder typified by generalized thickening of skin, mucosae, and certain viscera. Classic features include beaded eyelid papules and laryngeal infiltration leading to hoarseness. The disorder is clinically heterogeneous, with affected individuals displaying differing degrees of skin scarring and infiltration, variable signs of hoarseness and respiratory distress, and in some cases neurologic abnormalities such as temporal lobe epilepsy. Histologically, there is widespread deposition of hyaline (glycoprotein) material and disruption/reduplication of basement membrane.


65. A 40 year old man with acute promyelocytic leukemia develops scrotal ulcers 1 month after starting therapy. All tests for STD causes of ulcers as well as CMV and EBV are negative. What is the most likely cause?
A. All-trans retinoic acid
B. Daunorubicin
C. Methotrexate
D. 6-mercaptopurine
E. Cytarabine 

All-trans-retinoic acid (ATRA) has been shown to improve the outcome of patients with acute promyelocytic leukemia (APL). However, various adverse effects of ATRA treatment have been noted, such as scrotal and genital ulcers. 

 

he association of Port-wine stains on a limb with soft tissue swelling with or without bony over growth is:
A. Sturge-Weber syndrome
B. Goldenhar's syndrome
C. Klippel-Trenaunay syndrome
D. Bannayan-Riley-Ruvalcaba syndrome
E. Proteous syndrome 

he association of port-wine stain on a limb with soft tissue swelling with or without bony overgrowth is Klippel-Trenuanay syndrome. Klippel-Trenaunay syndrome is characterized by the Triad of port-wine malformations in association with deep venous system malformations, superficial varicosities, and bony and soft tissue hypertrophy. Sturge-Weber syndrome has 2 essential components: Facial port-wine stain and homolateral leptomeningeal angiiomattosis.


In addition to Kaposi’s sarcoma, HHV-8 infection is also the causative agent in:
A. Pyogenic granuloma
B. Pityriasis rosea
C. Angiosarcoma
D. Primary effusion lymphoma
E. Bacillary angiomatosis 

Human herpes virus 8 is a double stranded DNA virus. It is thought to be pathogenic in Kaposi’s sarcoma, primary effusion lymphoma, and Castleman’s disease. Primary effusion lymphoma is a rare B-cell lymphoma seen predominantly in patients with AIDS


Side effects from this antihistamine include gynecomastia, impotence, and loss of libido:
A. Fexofenadine
B. Promethazine
C. Cyproheptadine
D. Cimetidine
E. Doxepin 

Cimetidine, an H2 antihistamine, also competitively inhibits dihydrotestosterone at the androgen receptor site, with resultant antiandrogen side effects including gynecomastia, impotence, and loss of libido


You review a 70-year-old lady referred with an ulcer on her leg. On examination there is a 7 cm shallow ulcer overlying the medial malleolus with surrounding haemosiderin deposition and mild pitting oedema. The ulcer has been present and slowly enlarging over the past 12 months. What is the most appropriate course of action: 
A punch biopsy and referral for radiotherapy 
B arterial dopplers and refer to a vascular surgeon
C arterial dopplers and compression therapy 
D swab the ulcer and start antibiotics 
E incisional biopsy and start corticosteroid 

C. In this question the patient has a typical venous ulcer; it is large and shallow and overlies the medial malleolus. There is other evidence of venous insufficiency with haemosiderin deposition and pitting oedema. Venous ulceration is treated with compression and dressings, arterial dopplers are needed prior to compression therapy to check that the arterial supply is not compromised.


 Weekly CD4 T-cell counts are recommended for psoriasis patients treated with which biologic agent?
A. Etanercept
B. None of the above
C. Efalizumab
D. Alefacept
E. Infliximab 

Alefacept eliminates activated memory T-cells, so weekly CD4 T-cell counts are recommended


In which of the following Genodermatoses would one find cutaneous hyperpigmentation, blue lunulae and Kayser-Fleishcher rings
A. Hemochromatosis
B. Osteogenesis Imperfecta
C. Marfan's Disease
D. Gaucher's Disease
E. Wilson's Disease 

In Wilson's disease (Hepatolenticular Degeneration) one will find a vague greenish discoloration of the skin on the face, neck, and gentalia Hyperpigmentation), azure lunulae (sky-blue moons) of the nails, and Kayser-Fleischer rings. This is due to the body retaining excessive amounts of copper


Which of the following is not a side effect of bleomycin?
A. Flagellate hyperpigmentation 
B. Serpentine supravenous hyperpigmentation
C. Pulmonary fibrosis 
D. Acrosclerosis 
E. Erythromelalgia 

All of the above are side effects of bleomycin except serpentine supravenous hyperpigmentation. This is a side-effect of 5-fluorouracil


You are asked to review an inpatient with myelofibrosis and a rash, the admitting team are concerned about vasculitis. On examination of the distal limbs there is a non-blanching rash consisting of multiple 1–2 mm diameter flat red-purple lesions. What is the likely diagnosis:
A microscopic polyangiitis
B thrombocytopenia related petechiae
C polyarteritis nodosa
D meningococcal septicaemia
E vitamin K deficiency 

B. Thrombocytopenia related petechiae

The rash described consists of flat red-purple macules less than 4 mm
in diameter, this is a macular petechial rash. The commonest causes of
petechial rashes are platelet problems, in this case thrombocytopenia
related to myelofibrosis.

These are the most common changes in nail psoriasis:

Pitting of the nails – the surface of the nail develops small pits, looking rather like the surface of a thimble. The number of pits can vary from one to dozens.
Onycholysis – the nail becomes detached from the underlying nail bed and a gap develops under the nail. When it starts there is a white or yellowish patch at the tip of the nail, and this then extends down to the cuticle. The gap between the nail and the nail bed can become colonised by particular bacteria, such as pseudomonas, which can then produce a dark green pigment. The nail can become infected and discoloured and can easily be mistaken for melanoma under the nail.
Subungual hyperkeratosis – a chalky substance accumulates under the nail. The nail becomes raised and can become tender, especially when the surface of the nail is pressed. Subungual hyperkeratosis of the toenails can be particularly uncomfortable because when wearing shoes the nail may be put under constant pressure.
Discolouration – this may be seen as unusual nail colouration, such as yellow-brown.
Onychomycosis (on-ik-o-mi-ko-sis) – a fungal infection that can cause thickening of the nails. This could be present alongside nail psoriasis and can be confused in diagnosis. If diagnosed correctly it can be treated with systemic antifungal medication. It is estimated that approximately 35% of people who have nail psoriasis may also have a fungal infection that could make the nails worse. Treating the fungal infection may not have any effect on the clearance of nail psoriasis. 

Apocrine glands are found in all of the following areas of the body except:
A. Axillae
B. Perineum
C. Breasts
D. Eyelid
E. Palms 

Apocrine glands operate by decapitation secretion and are activated by epinephrine and norepinephrine. They are located in a few distinct areas of the body, which include axillae, anogenital region, Moll’s glands of the eyelids, mammary glands of the breast and the ceruminous glands of the external auditory canal.


Ectopia lentis (downward displacement of the lens) is characteristic of:
A. Multiple Carboxylase deficiency
B. Homocystinuria
C. Ehlers-Danlos syndrome
D. Phenylketonuria
E. Marfan syndrome 

Ectopia lentis (downward displacement) is seen in homocystinuria. Upward displacement is seen in Marfan syndrome. There are no changes in the lens in phenylketonuria or multiple carboxylase deficiency


Weibel-Palade bodies are seen in:
A. Spitz Nevi
B. Endothelial cells 
C. Cells infected with MCV
D. Plasmacytoid Cells
E. Malakoplakia 

Weibel-Palade bodies are seen in endothelial cells and are therefore found in vascular lesions. Kamino bodies are found in Spitz nevi. Henderson Patterson bodies are seen in molluscum. Dutcher bodies are intranuclear inclusions seen in plasmacytoid cells. Michaelis Gutmann bodies are partially digested bacteria seen in malakoplakia


You see a patient with chronic scarring alopecia. On examination they also have fixed, indurated, erythematous papules and plaques on the face and ears. There are also areas of scarring, hyperpigmentation and hypopigmentation. On the scalp there is scale and keratotic plugging of the hair follicles. You suspect a diagnosis of discoid lupus, what is the chance of this patient developing systemic lupus: 
A 100% by 10 years 
B 100% by 6 months 
C 50% 
D 5% 
E none 

This patient has the typical signs of discoid lupus (DLE), a chronic photoexacerbated autoimmune disorder. In answer to the question approximately 5% of patients with DLE will go on to develop systemic lupus (SLE); although around 25% with SLE will develop DLE-like skin lesions. DLE is treated with topical steroids, anti-malarial drugs, photoprotection and in resistant cases immunosuppression.


The treatment of choice for Oroya Fever is:
A. Chloramphenicol
B. Erythromycin
C. Penicillin
D. Minocycline
E. Doxycycline

The treatment of choice for Oroya Fever is chloramphenicol because of frequent superinfection with Salmonella

 

The characteristic lesion of keratoacanthoma is a dome-shaped, flesh-colored, crateriform or “volcano-like” tumor with raised edges and a central keratin horn. Keratoacanthomas usually range from 1 to 2 cm in diameter and develop unusually rapidly over 2 to 8 weeks. Commonly appear on sun-exposed areas
such as the limbs, head and neck. KAs may spontaneously regress over 6 to 12 months and leave a depressed annular scar. skin biopsy showing a keratin-filled central crater with infiltrating, well-differentiated atypical squamous nests. Treatment first line: Surgical excision with clear margins. Second line: Cryotherapy, intralesional 5-fluorouracil (5-FU) or methotrexate for nonsurgical candidates  

 

All of the following are features of scarlet fever except:
A. S. aureus infection
B. Pastia’s lines
C. Pharyngitis
D. Circumoral pallor
E. Sandpaper-like texture 

  

A. Scarlet fever is primarily a disease of children with most cases occurring between the ages of 1 and 10 years of age. Streptococcus pyogenes is the causative organism. It produces the streptococcus pyrogenic exotoxin which elicits the cutaneous manifestations by enhancing delayed type hypersensitivity to streptococcal products. Clinical findings include fever, sore throat, headache, chills, sandpaper rash on the trunks, strawberry tongue, and Pastia’s lines (linear petechial streaks found in flexural locations

 

89. Which syndrome is characterized by hyperhidrosis, lack of pain sensation, hypersalivation, and absent fungiform papillae?
A. Rubinstein-Taybi syndrome
B. Noonan Syndrome
C. Turner Syndrome
D. Riley-Day Syndrome 
E. Cornelia de lange Syndrome 


Riley-Day syndrome is also known as Familial Dysautonomia. It is an autosomal recessive disorder with the gene defect on the long arm of chromosome 9. Patients have unmyelinated sensory and sympathetic neurons and autonomic dysfunction, leading to hyperhidrosis, decreased corneal sensation and tear flow, hypersalivation, gastroesophageal reflux, decreased deep tendon reflexes, and lack of pain sensation. They also exhibit abnormal histamine skin test.

 

The most specific marker of drug-induced lupus is:
A. Anti-La Ab
B. ANA
C. Anti-ds DNA Ab
D. Anti-histone Ab 
E. Anti-Ro Ab 

  

Anti-histone Ab is most specific for drug-induced lupus. In addition to minocycline, hydralazine, procainamide, isonaizid (INH), penicillamine and anti-convulsants have been associated with drug-induced lupus-like syndrome

 

Which of the following dermal cells always express CD11c and CD6?
A. Mononuclear phagocytic cells
B. Glomus cells
C. Dermal dendrocytes
D. Fibroblasts
E. Mast cells 


A. The mononuclear phagocytic cell includes monocytes, macrophages and dermal dendrocytes. All phagocytic skin macrophages express CD11c and CD6. Fibroblasts do not produce a CD marker. Mast cells are derived from bone marrow-residing CD34+ stem cells. They do not produce CD11c or CD6. Dermal dendrocytes do not express these marker either.

 

You are asked to review a 70-year-old lady with longstanding rheumatoid arthritis. She has developed a number of painful haemorrhagic lesions on the finger pads. What is the most likely diagnosis: 
A Libman-Sacks endocarditis 
B drug reaction 
C Bywater’s lesions 
D Janeway’s lesions of subacute streptococcal endocarditis 
E Osler’s nodes of subacute staphylococcal endocarditis. 

 

The description and history in this question is consistent with Bywater’s lesions a cutaneous manifestation of rheumatoid arthritis. They are cutaneous infarcts related to small vessel rheumatoid vasculitis. In addition they may also present under the nails as painless red-brown lesions mimicking the splinter haemorrhages of subacute bacterial endocarditis (SBE). Libman-Sacks endocarditis is a sterile endocarditis which occurs in patients with systemic lupus. Osler’s nodes are painful, red, raised lesions on the palms and soles which may be associated with SBE. Janeway’s lesions are small asymptomatic maculo-nodules on the palms or soles and are pathognomonic of SBE.

 

Which of the following is the most common adverse effect of Thalidomide therapy?
A. Diarrhea
B. Hypertension
C. Photosensitivity
D. Skin discoloration
E. Sedation 

 

E. Thalidomide was introduced in the late 1950's as a "safe" sleeping aide. It readily penetrates the CNS, where it exerts a hyposedative effect comparable with barbiturates. By far, the most common adverse effect from thalidomide is sedation, which in many patients may require that primarily night-time doses be utilized.



95. Blueberry Muffin Baby can be a feature of all of the following EXCEPT:
A. Rhabdomyosarcoma
B. Langerhans’ cell histiocytosis
C. TORCH
D. Hemolytic disease of the newborn
E. Klippel-Trenaunay-Weber syndrome 

MCQ Dermatology Blueberry muffin lesions can be seen in the setting of prenatal infections (e.g. TORCH), severe anemia (e.g. Hemolytic Disease of the newborn) and neoplastic diseases (e.g. rhabdomyosarcoma). It is not associated with the Klippel-Trenaunay-Weber syndrome 

Which of the following statements best describes the pathogenesis of scrofuloderma?
A. Autoinoculation of M. tuberculosis from advanced pulmonary tuberculosis
B. Hematogenous spread of M. tuberculosis from a distant site of infection
C. Primary incoculation of M. tuberculosis in a sensitized host
D. An cutaneous manifestation of military tuberculosis
E. Contiguous spread from an underlying focus of tuberculous lymphadenitis 

E. Scrofuloderma is the result of contiguous spread onto skin from an underlying focus of tuberculous infection. This typically occurs in a sensitized host with low immunity

 



Urticaria pigmentosa is linked to a defect in the c-kit protooncogene. What autosomal dominant skin disease also has been linked to this defect?
A. Piebaldism
B. Hypomelanosis of Ito
C. Incontinentia pigmenti
D. Waardenburg syndrome
E. Hermansky-Pudlak syndrome 

 

Piebaldism is linked to a defect in the c-kit protooncogene.

 

Common causes of drug-induced linear IgA:
A. Vancomycin
B. Penicillin
C. Cephalosporins
D. Captopril
E. All of these answers are correct 

MCQ Dermatology E Linear IgA disease that is induced by drugs is most commonly secondary to vancomycin. Other implicated drugs include other antibiotics and captopril.

 

The usual culture medium for Mycobacterium leprae is:
A. Chocolate agar in 10% CO2
B. Lowenstein-Jensen
C. It cannot be cultured
D. Sheep’s blood agar
E. Agar supplemented with heme and nicotinamide 

M. leprae cannot be grown in media or cell culture. It has been grown in mice footpads and in
armadillos. Humans are the only natural host. It is endemic in the tropics and found worldwide.  

 

 

100. Which of the following is a ssDNA virus:
A. Parvovirus 
B. Parapox
C. Herpesvirus
D. Picornovirus
E. Adenovirus 

Parvovirus is the only ssDNA virus listed 

 

101. What is the average duration of the telogen cycle in terminal scalp hair?
A. 9 months
B. 6 months
C. 2-3 weeks
D. 2-6 years
E. 3 months

 

E. The average duration of the telogen phase of the hair cycle is 3 months; this feature explains why telogen effluvium is typically observed 3 months following a traumatic event or serious illness. The average duration of the anagen phase of the hair cycle is 2-6 years, whereas that of the catagen cycle is 2-3 weeks.

 

. Apocrine glands
A. demonstrate decapitation secretion. 
B. are fully functional at birth.
C. are thermoregulatory.
D. are diffusely distributed on the body.
E. demonstrate holocrine secretion 

 

Apocrine glands show decapitation secretion. Like eccrine glands, apocrine glands are composed of three segments, the intraepidermal duct, the intradermal duct, and the secretory portion. The duct of the apocrine gland usually leads to a pilosebaceous follicle above the entrance of the sebaceous duct. Apocrine glands are found in the axillae, anogenital region, external ear canal (ceruminous glands), in the eyelids (Moll’s glands), and in the breast (mammary glands). Apocrine glans are functional only at puberty. Their initial secretion is odorless. Hidradenitis suppuritiva is a disease involving apocrine glands.



Naxos syndrome is characterized by a right sided cardiomyopathy, wooly hair, and keratoderma. The epidermal structure defective in Naxos syndrome is:
A. desmoplakin
B. plakoglobin
C. desmoglein 3
D. desmoglein 1
E. Keratin 1/10 

 

Plakoglobin is an intracellular desmosomal component which binds desmogleins/desmocollins on one side and to desmoplakin on the other. Desmoplakin in turn binds to the keratin intermediate filaments, K1/10 in most cases. Mutation of desmoplakin leads to CarvajaL syndrome, which is associated with a striate palmoplantar keratoderma, woolly hair and Left sided cardiomyopathy. A simple way to remember this is the L in Carvajal cooresponds to the Left sided cardiomyopathy vs. the right sided disease in Naxos disease.

 

Which of the following is a true statement about infliximab?
A. FDA-approved for the treatment of psoriasis
B. Not effective in psoriatic arthritis
C. Increases C-reactive protein
D. Chimeric monoclonal antibody against TNF-alpha
E. Indicated for mild to moderate psoriasis  

 

D. Infliximab (brand name Remicade) is a chimeric recombinant fusion protein composed of a human TNF-alpha receptor with IgG1-Fc. The immunoglobulin portion is derived from a human constant region and a murine variable region. Infliximab neutralizes both soluble and transmembrane TNF-alpha. This medication has been FDA approved for the treatment of Crohn’s disease, rheumatoid arthritis and ankylosing spondylitis

 

 

.  Which virus is most commonly associated with oral hairy leukoplakia?
A. Human papilloma virus
B. Epstein-Barr virus
C. Parvovirus
D. Pox virus
E. Herpes virus 

 

Oral hairy leukoplakia is an oral mucosal infection most often caused by the Epstein-Barr virus, which occurs in immunocompromised patients. Clinically, thick, white plaques are noted on the lateral sides of the tongue, often fissured.

 

Which of the following diseases with immunodeficiency has an increased risk of lymphoreticular malignancy?
A. Chronic granulomatous disease
B. Severe combined immunodeficiency syndrome
C. Leiner’s disease
D. Wiskott-Aldrich syndrome
E. Job syndrome 

 

Wiskott Aldrich syndrome (WAS) is a disease with immunological deficiency and reduced ability to form blood clots. Signs and symptoms include easy bruising or bleeding due to a decrease in the number and size of platelets; susceptibility to infections and to immune and inflammatory disorders; and an increased risk for some cancers (such as lymphoma). Also, a skin condition known as eczema is common in people with WAS. Wiskott Aldrich syndrome is caused by mutations in the WAS gene and is inherited in an X-linked manner. It primarily affects males. Treatment may depend on severity and symptoms in each person, but hematopoietic cell transplantation is the only known cure. Hematopoietic cells are the blood-forming stem cells that can be found mainly in the sponge-like material found inside bones (bone marrow), but also in the bloodstream (peripheral blood stem cells (PBSCs), and in the umbilical cord. Prognosis have improved over time due to better management of the disease. People who have a successful and uncomplicated hematopoeitic cell transplantation, usually have normal immune function and, normal survival.

 

 

108. he vector of Trench Fever is the:
A. Cat flea (Ctenocephalides felis)
B. Human body louse (Pediculus humanus corporis)
C. Trombiculid mite
D. Sandfly (Phlebotamus perniciosus)
E. Rat flea (Xenopsylla cheopis) 

The human body louse (Pediculus humanus corporis)is the vector of Trench Fever, Epidemic Typhus, and Relapsing Fever



Cicatricial pemphigoid antibodies directed against this are associated with high frequency of malignancy:
A. Laminin 5 
B. Beta4-integrin
C. BPAg2
D. Laminin 6
E. All of these answers are correct 

 

Anti-laminin 5 cicatricial pemphigoid (CP) is also known as anti-epiligrin CP. Anti-epiligrin CP is associated with an increased frequency of internal adenocarcinomas. Laminin 5 is composed of three chains (heterotrimer), alpha3, beta3, gamma2. Antibodies are frequently directed against the alpha3 chain, and so cross-reactivity can be observed with laminin 6, as laminin 6 (alpha3beta1gamma1) has the alpha3 chain as well. Beta4-integrin antibodies have been associated with ocular CP. BPAg2 antibodies are seen in CP patients that have mucosal as well as skin disease

 

Tricomycosis axillaris is caused by:
A. Burkholderia mallei
B. Corynebacterium tenuis 
C. Proteus species
D. Micrococcus sedentarius
E. Corynebacterium minitissimum 

 

Trichomycosis axillaris (TMA), also known as Trichobacteriosis, is a superficial infection of the axillary hair shaft caused by the aerobic gram positive bacterium Corynebacterium flavescens, formerly named Corynebacterium tenuis. This bacteria is present in nature as both bacillus and diphtheroid, and mostly causes disease in humid, tropical climates .

 

A patient with a blue-red discoloration of the nail plate reports that the same finger becomes very tender when exposed to the cold. You suspect a:
A. Pyogenic granuloma
B. Glomus tumor
C. Periungual verruca
D. Mucous cyst
E. Pterygium 

 

Glomus tumors are tumors of the arterio-venous anastamosis of the digital dermis. They occur most frequently in the nail bed. The commonly have a bluish-red discoloration and may be tender or painful with exposure to heat or cold

 

 In the epidermis, the cell most responsible for antigen detection and processing is the
A. Melanocyte
B. Merkel Cell
C. Langerhans cell 
D. CD4+ T cell
E. Keratinocytes 

 

The Langerhans cell is a bone-derived, antigen-presenting cell found in all layers of the epidermis, oral mucosa, esophagus, and vagina. Langerhans cells ingest and process antigens, mature, migrate to a local lymph node, and then present the antigen to a naïve (or resting) T cell, activating that T cell. The Langerhans cell is central to the pathogenesis of atopic dermatitis, psoriasis, allergic contact dermatitis, and certain infections, such as Leishmaniasis.

 

50-year-old is admitted to coronary care after suffering an anterior myocardial infarction. After thrombolysis with streptokinase the patient is started on aspirin, clopidogrel, metoprolol, ramipril and simvastatin. During his recovery you are asked to see the patient as he has developed a rash. On examination he has multiple small beefy red plaques with silvery scale most prominent on the extensor surfaces. The patient’s identical twin brother has psoriasis. What is the most likely diagnosis: 


A He has caught psoriasis from another patient on the ward 
B Latent psoriasis precipitated by beta-blocker 
C Psoriasiform drug reaction to aspirin 
D Latent psoriasis precipitated by ACE inhibitor 
E Latent psoriasis precipitated by streptokinase 

 

B. Triggering events that can precipitate latent psoriasis: •cutaneous injury – the Koebner phenomenon •infections – particularly streptococci, HIV •psychogenic stress •endocrine factors – pregnancy, hypocalcaemia •drugs – beta-blockers, lithium, anti-malarial drugs, interferon. In this question the likely precipitants are the psychogenic stress of the myocardialinfarction and beta-blockers. Psoriasis is not infectious and cannot be caught. The other drugs postulated are not known precipitators of psoriasis.

 

 

Contagious pustular dermatitis (Orf) is caused by a:
A. Poxvirus 
B. Gram-positive spore-forming rod
C. Papovavirus
D. Herpesvirus
E. Paramyxovirus 

 

Orf is caused by orf virus (OV), a poxvirus of the genus Parapoxvirus

 

The most appropriate treatment of a 2 month-old with a hemangioma involving the lateral canthus is:
A. Observation and reassurance of the parents
B. Pulsed dye laser
C. Surgical excision
D. Interferon-alpha
E. Systemic corticosteroids 

 

E. Since hemangiomas characteristically follow a course of proliferation followed by spontaneous involution, many of the lesions can be followed with conservative management including observation. However, indications for treatment of hemangiomas include obstruction of vital function, high-output cardiac failure, ulceration, infection, diaper area location and location on the face. Aggressive treatment is indicated for a hemangioma located on the lateral canthus since two of the above indications apply. Importantly, obstruction of the visual field may impair development of the visual cortex in an infant. Oral glucocorticoids are the mainstay of treatment for hemangiomas, with 30 to 60% of lesions responsive to therapy. Alternative treatments for problematic hemangiomas include intralesional steroids, interferon alpha-2a, and pulsed dye laser

 

 Human sebum is distinguished from lipids of internal organs by the presence of:
A. Squalene
B. Wax esters
C. Glycerides
D. Cholestrol
E. Cholestrol esters

 

 

B. As human sebum exits the sebaceous gland, its major constituents are squalene, cholesterol, cholesterol esters, triglycerides, and wax esters. With passage through the hair follicle, triyglycerides in the sebum become hydrolyzed by bacterial enzymes, so that by the time the sebum reaches the skin surface, it contains free fatty acids, mono- and diglycerides in addition to the original components. Human sebum is distinguished by the presence of wax esters and squalene. The lipids of human internal organs contain no wax esters and little squalene. The squalene that is produced in internal organs is quickly converted to lanosterol and then to cholesterol, so it does not remain in its original form. Human sebaceous glands do not convert squalene to sterols. 

 

 

Methemoglobinemia is an adverse reaction to treatment with which agent?
A. Plaquenil
B. Dapsone
C. Methotrexate
D. Cyclophosphamide
E. Azathioprine 

 

Dapsone is a lipid-soluble sulfone drug that is used widely in dermatology in a variety of conditions including dermatitis herpetiformis, leprosy, and neutrophilic dermatoses. Adverse effects from dapsone are both pharmacologic and idiosyncratic and include hemolytic anemia, methemoglobinemia, agranulocytosis, hypersensitivity syndrome and neuropathy. Of these, the first two are pharmacologic and anticipated, to some degree, in most patients treated with dapsone. However, the magnitude of toxicity varies greatly among individuals on the drug. Methemoglobinemia is the formation of methemoglobin in the blood, which has a decreased oxygen-carrying capacity compared with hemoglobin and can result in cyanosis. The reaction is related to the N-hydroxy metabolites of dapsone, which are potent oxidants. G6PD-deficient individuals are more susceptible to oxidative stresses, including those from dapsone metabolites, and a baseline G6PD level is recommended prior to initiation of dapsone therapy. In the event of emergent methemoglobinemia, oral methylene blue (100-300 mg/day) can be used to decrease methemoglobin levels. However, if the patient is G6PD deficient, this strategy is ineffective  

 

 

Which form of cutaneous tuberculosis is associated with the highest level of immunity?
A. Tuberculosis gumma
B. Tuberculosis verrucosa cutis
C. Lupus vulgaris
D. Miliary tuberculosis
E. Tuberculosis chancre 

 

Lupus vulgaris is an chronic, progressive form which usually occurs around nose, neck and earlobes, notable for gelatinous "apple-jelly" hue. Tuberculosis chancre and tuberculosis verrucosa cutis cutis are primary inoculations of tuberculosis infection. Tuberculosis gumma and miliary tuberculosis are associated with lowered states of immunity 

 

Which mechanoreceptor found in hair bearing areas sense deep touch and vibration?
A. Krase end-bulb
B. Vater-Pacini corpuscle
C. Merkel cell
D. Meissner corpuscle
E. Free nerve ending 

 

Vater-Pacini corpuscles sense deep touch and vibration. Merkel cells are slow adapting type I mechanoreceptors found among basal keratinocytes. Meissner corpuscles are found in the dermal papilla, especially in the palms and soles. Krase end-bulbs are mucocutaneous end-organs found on the glans penis, prepuce, clitoris, labia minora, and vermillion border of the lip 

 

Premalignant leukoplakia of the oral mucosa is associated with
A. Werner Syndrome
B. Bloom syndrome
C. Rothmund-Thomson syndrome
D. Dyskeratosis Congenita
E. Xeroderma Pigmentosum 

 

Dyskeratosis Congenita (also known as Zinsser-Engman-Cole syndrome) is thought to have two modes of inheritance. The more common X-linked disorder is due to a mutation in the Dyskerin gene, while the autosomal dominant form is due to a mutation in TERC, a telomerase RNA component. Clinical features include reticulated gray-brown hyperpigmentation, paloplantar hyperkeratosis, alopecia, onychodystrophy, premalignant leukoplakia of any mucosal surface, and mental retardation 

 

123: Measles is caused by a:
A. Togavirus
B. Rhabdovirus
C. Picornavirus
D. Paramyxovirus
E. Adenovirus 

 

The measles virus is an RNA virus belonging to the Paramyxovirus family 

 

A halo nevus is a melanocytic nevus with a surrounding well-circumscribed annulus of hypo- or depigmented skin . Halo nevi can be solitary or multiple and generally affect individuals under the age of 20 years. In general, those patients with halo nevi have an overall increased number of melanocytic nevi. Although most pigmented lesions with halos are benign, malignant melanoma can rarely be seen with an associated halo. If a pigmented lesion has an irregular border and halo or shows other atypical features, it should be biopsied. 

 

In addition to minocycline, which of the following drugs has been associated with drug-induced lupus erythematosus-like syndrome?
A. Rifampin
B. Hydralazine 
C. Itraconazole
D. Amiodarone
E. Doxycycline 

 

B. Drugs associated with drug-induced SLE include minocycline, hydralazine, procainamide, isonaizid, penicillamine, and anti-convulsants 

 

What is the causative agent of this parasitic eruption which also causes patchy pulmonary infiltrate and eosinophilia?
A. Filariasis
B. Dracunculiasis
C. Larva migrans
D. Larva currens
E. Gnathostomiasis 

 

Larva migrans is caused by Ancylostoma braziliense, a hookworm of cats and dogs. It also associated with Loeffler's syndrome, patchy infiltrate of the lungs with eosinophilia. Treatment for this parasitic infection is ivermectin or albendazole   

 

What is the most abundant collagen found on fetal skin?
A. Type III Collagen
B. Type II Collagen
C. Type IV Collagen
D. Type VII Collagen
E. Type I Collagen 

 

Type III collagen is found in the fetal skin. It is also present in the gastrointestinal tract, blood vessels, and the basement membrane. A defect in this collagen results in the Ehlers-Danlos, vascular type.  

 

 Which of the following markers are specific and reliable for Merkel cells?
A. CD34
B. CK20
C. CD20
D. HMB-45
E. CD3 

 

Cytokeratin (CK) 20 is a reliable marker for Merkel cells. CD20 is a marker for B-cells and CD3 is a T-cell marker. HMB-45 is used in staining for immature melanosomes and is reactive in melanoma. CD34 is used to stain dermatofibrosarcoma protuberans. 

 

 In a child with zinc deficiency, yet normal or near normal zinc levels, which test could be a valuable adjunctive test?
A. Niacin
B. Iron
C. Alkaline phosphatase
D. Magnesium
E. Manganese  

 

 

he diagnosis of zinc deficiency should be consideredi in at-risk individuals with acral or periorificial dermatitis. Chronic diaper dermatitis in an infant should lead to the evaluation for zinc deficiency. A low serum zinc level can usually confirm the diagnosis. If normal or near normal a low serum alkaline phosphatase, a zinc-dependent enzyme, may be a valuable adjunctive test.

The risk of pseudotumor cerebri in patients taking isotretinoin is increased by:
A. Dehydration
B. Comorbid affective disorder
C. Doses higher than 1.0 mg/kg/day
D. Concomitant use of tetracycline 
E. Concomitant use of TMP-SMX 

 

The risk of pseudotumor cerebri is increased in patients on isotretinoin and a tetracycline 

 

131: How long is the life cycle for the mite that causes scabies ?
A. 1 day
B. 1 month
C. 6 months
D. 1 year
E. 1 week 

 

Scabies is caused by the mite Sarcoptes scabiei var. hominis. The life cycle of the mite is 30 days. A female mite will lay 60-90 eggs during her life 

 

131: An 81-year-old woman presents with a rapidly growing lesion on her right helix. On examination there is a 2 cm diameter, well defined, skin coloured nodule with a central keratin plug. You suspect a diagnosis of keratoacanthoma. What treatment option should you recommend: 
A Radiotherapy 
B Punch biopsy 
C Incisional biopsy 
D Excision biopsy 
E Curettage and cautery 

 

Keratoacanthomas are common skin tumours that occur on sun exposed sites. They occur suddenly and expand rapidly up to 2 cm in diameter over a few weeks. They clinically appear as a well defined nodule with a central keratin plug. After a number of weeks they spontaneously regress without sequelae. Pathologically and clinically they are similar in appearance to well differentiated SCCs with significant cytological atypia and a high mitotic rate. For a pathologist to confidently diagnose keratoacanthoma they need to examine the whole specimen to account for the overall architecture of the lesion. It is thought that up to 10% of lesions diagnosed as keratoacanthomas are actually well differentiated SCCs and many dermatologists feel that all keratoacanthomas should be treated as SCCs. the patient should undergo an excision biopsy with minimal surgical margins.  

 

Retinal hemangioblastomas are found in which syndrome:
A. Kasabach-Merritt syndrome
B. Klippel-Trenaunay Weber syndrome
C. Sturge-Weber syndrome
D. Von-Hippel Lindau disease
E. Osler-Weber-Rendu disease  

 

Von Hippel-Lindau syndrome is an autosomal dominant condition caused by a defect in the VHL tumor suppressor gene. This disease is characterized by retinal hemangioblastomas, often resulting in visual impairment and blindness if left untreated. In addition, many tumors are seen including pheochromocytoma, renal cell carcinoma, and hemangioblastomas of the cerebellum, medulla, and spinal cord. Pancreatic and renal cysts are also a feature of this condition. Finally, polycythemia can occur as a result of erythropoietin production by renal cell carcinoma. Von Hippel-Lindau syndrome is a progressive, universally fatal condition which presents most often in the fourth decade of life  

 

134”: Which of the following is the vector for Dengue fever?
A. Lutzomyia verrucarum
B. Phlebotomus papatasii
C. Xenopsylla cheopis
D. Aedes aegypti
E. Culex mosquito 

 

Dengue fever also known as "break-bone" fever is characterized by sudden high fever, backache, retro-orbital pain, bone/joint pain, weakness, and malaise. It is caused by an arbovirus (RNA virus) which is transmitted by Aedes aegypti, a species of mosquito.  

 

 

136: The risk of fetal death with intrauterine parvovirus infection may occur with infection in which trimester:
A. First
B. Third
C. Second
D. First, Second and Third 
E. None of these answers are correct 

 

Fetal hydrops may occur with parvovirus infection during all three trimesters although the greatest risk is during the second trimester. Congenital anomalies are not a feature 

 

137: Lipomembranous change is seen in:
A. Sclerosing panniculitis 
B. Sebaceous carcinoma
C. Cystic sebaceous adenoma
D. Hibernoma
E. Mucocele  

 

Lipomembranous change is a non-specific histologic pattern that is most commonly seen in lipodermatosclerosis, which is also known as sclerosing panniculitis; this condition may be secondary to venous stasis  

 

138: Desmosine and isodesmosine are typical amino acids found in:
A. Collagen fibers
B. Anchoring plaques
C. Anchoring fibril
D. Elastic fibers
E. Heparan sulfate 

 

Desmosine and isodesmosine are typical amino acids found in elastic fibers. They crosslink fibrillin. Anchoring fibrils are composed of collagen VII and collagen fibers and have the most typical amino acids of proline and hydroxyproline. Heparan sulfate do not typically contain these amino acids 

 

139: Which of the following is the most common long term sequelae from congenital rubella syndrome?
A. Nystagmus
B. Macrocephaly
C. Microcephaly
D. Deafness 
E. Saber shins 

 

Deafness may occur in up to 50% of infants with congenital rubella syndrome 

 

140:  Which of the following make up the major protein of the cornified cell envelope?
A. Filaggrin
B. Loricrin
C. Involucrin
D. Laminin V
E. Envoplakin 

 

Loricrin is the major protein component of the cornified cell envelope (CE). Involucrin is cross-linked by transglutaminae in the granular layer to form an insoluble cell boundary. Envoplakin may link the CE to desmosomes and to kertin filaments. Filaggrin is thought to promote aggregation and disulfide bonding of keratin filaments in CE. It is degraded into urocanic acid and pyrrolidone carboxylic acid. Both of which hydrate the stratum corneum and block UV radiation. Laminin V is found in the basement membrane and is not involved in the formation of the cornified cell envelope. 

 

141: A 52-year-old man presents with gynaecomastia. On examination he has symmetrical enlargement of both breasts with no evidence of any masses or suspicious lesions. The patient takes a number of medications and you suspect one may be responsible, which of the following is the most likely culprit: 
A Furosemide 
B Cimetidine 
C Ranitidine 
D Atorvastatin 
E Phenytoin 

 

Gynaecomastia in a male is often due to an imbalance of the oestrogens and androgens. Unilateral gynaecomastia is suspicious of underlying malignancy and should be investigated thoroughly. Obesity may cause a pseudo-gynaecomastia due to adipose tissue deposition. Cimetidine is the most likely cause of the patient’s gynaecomastia. 

 

142: A 40-year-old man presents with recurrent mouth ulcers. Which of the following is not a recognised cause of mouth ulceration: 
A Herpes simplex infection 
B Iron deficiency 
C Congenital syphilis 
D Nicorandil 
E Methotrexate in overdose 

 

Causes of mouth ulceration: 
•aphthous, idiopathic 
•traumatic 
•herpes simplex infection 
•B12 deficiency, iron deficiency 
•coeliac disease, inflammatory bowel disease 
•Reiter’s syndrome 
•immunodeficiency 
•nicorandil, methotrexate, chemotherapy 
•oral cancers, metastatic cancer 
•syphilitic snail track ulcers 
•inflammatory dermatosis e.g. pemphigus. In this question congenital syphilis is not a cause of mouth ulceration, although secondary syphilis is. 

 

143: A child with phenylketonuria likely presents with which cutaneous problems?
A. Alopecia universalis
B. Generalized hypopigmentation
C. Leg ulcers
D. Blue-gray generalized hyperpigmentation
E. Generalized hyperpigmentation 

 

Phenylketonuria is an autsomal recessive disorder caused by a mutation on the long arm of chromosome 12. A deficiency of phenylalanine hydroxylase or its cofactor tetrahydrobiopterin leads to accumulation of phenylalanine. Clinical features include generalized hypopigmentation, eczematous dermatitis, sclerodermoid changes, seizures, psychomotor delay, urine with “mousy” odor, mental retardation  

 

145: Surgery, Parturition, Fever, Kwashiokor and Hypervitaminosis A are all causes of:
A. Telogen effluvium
B. Alopecia areata
C. Both telogen and anagen effluvium
D. Anagen effluvium
E. Androgenetic alopecia 

All of the listed stressors can induce telogen effluvium. Other causes include traction and some drug exposures. Usually will only involve up to 50% of scalp hairs and will resolve within 2-3 months 

 

146; A 2 day-old full term neonate develops blotchy erythematous macules with small central pustules over the upper trunk and extremities. A gram stain reveals predominantly eosinophils. What is the most likely diagnosis?
A. Incontinentia pigmenti
B. Erythema toxicum neonatorum 
C. Urticaria pigmentosa
D. Transient neonatal pustular melanosis
E. Miliaria  

 

Erythema toxicum neonatorum is a very common eruption in healthy newborns. A gram stain reveals sterile pustules containing eosinophils  

 

147: Petechial lesions at the junction of the hard and soft palate are characteristically seen in which disease?
A. Kawasaki’s disease
B. Infectious mononucleosis
C. Scarlet fever
D. Rocky Mountain spotted Fever
E. Rickettsialpox infection  

 

Forsheimer spots are distinctive pinhead size petechiae found at the junction of the hard and soft palate. They may be seen in infectious mononucleosis  

 

148: Cutaneous osteomas are seen in which syndrome?
A. Carney complex
B. Albright hereditary osteodystrophy 
C. Gaucher’s syndrome
D. Waardenburg syndrome
E. LEOPARD syndrome 

 

Albright hereditary osteodystrophy is caused by mutations in the Gs subunit of adenylate cyclase. There is calcification and ossification due to pseudohypoparathyroidism, absent 4th knuckle, and hypogonadism 

 

149: The treatment for acrodermatitis enteropathica is:
A. Vitamin B12 supplementation
B. Zinc supplementation
C. Phlebotomy
D. Iron supplementation
E. Vitamin B1 supplementation 

 

Acrodermatitis enteropathica is due to a defect in zinc absorption and will respond to zinc supplementation. Iron, Vitamin B1/12 supplementation will not result in improvement in this condition. Findings include periorificial, scalp, and acral dermatitis, scaling, vesicles/bullae, erosions, alopecia, diarrhea and stomatitis  

 

150: Hair follicle development in the human embryo begins during
A. Within 2 weeks of fertiization
B. 3rd trimester
C. At the blastocoele stage
D. 2nd trimester
E. 1st trimester 

 

Hair follicles begin in the third month of fetal life as a downgrowth of cells from the epidermis (3rd month=12th week=1st trimester) 

 

151: The active spectrum for cutaneous vitamin D3 synthesis is: 
A. 220-290nm 
B. 290-320nm 
C. 320-400nm 
D. 400-410nm 
E. Both First and Second Choice 

 

E. The source states that vitamin D3 synthesis occurs at wavelengths < 320 making choices 220-290nm and 290-320nm correct.

 

152: A child develops an allergic reaction at the site of an influenza vaccine. To which of the following substances may she be allergic to? 
A. Lanolin 
B. Thimerosol 
C. Ethylenediamine dichloride 
D. Triclosan 
E. Gluteraldehyde 

 

Thimerosol is a preservatives in vaccines such as the influenza, tetanus, and diphtheria vaccines. It is also found in antitoxins and immunoglobulins. Thimerosol is a mercury-containing organic compound. Lanolin is from the sebum of sheep. Ethylenediamine dichloride is a stabilizer in topical creams, medicines, dyes, insecticides, and fungicides and was previously found in nystatin cream. Triclosan is an antibacterial agent found in soap, shampoo and mouthwash. Gluteraldehyde is a cold sterilizing solution used for medical and dental equipment. 

 

153;  Which of the following is the most common initial site of metastasis from a primary BCC? 
A. Lungs 
B. Regional lymph nodes 
C. Bone 
D. Liver 
E. Pleura 

 

The metastatic potential of BCC is very low with rates ranging from 0.0028 to 0.1%. The head and neck region is the most frequent location of the primary tumor with regional lymph nodes being the most common site of metastasis. The lungs, bone, liver, and pleural are also potential sites of metastasis 

 

155: Darier's sign is described as: 
A. When an intact epidermis shears away from the underlying dermis, leaving a moist surface 
B. Spreading bulla phenomenon with pressure on an intact bulla 
C. Swollen, itchy and or red after stroking the skin 
D. Central depression within a lesion when squeezed along its margins 
E. Disappearance of color when the lesion is pressed 

 

Swollen, itchy and or red after stroking the skin is referred to Darier's sign and can be seen in systemic mastocytosis or urticaria pigmentosa. Nikolsky sign can be seen when an intact epidermis shears away from the underlying dermis, leaving a moist surface (seen in pemphigus vulgaris, staphylococcus scalded skin syndrome (SSSS), and toxic epidermal necrosis). Spreading bulla phenomenon with pressure on an intact bulla is referred to as Asboe-Hansen sign, commonly seen with pemphigus vulgaris. Central depression within a lesion when squeezed is referred to as the dimpling sign and is seen in dermatofibromas. Disappearance of color or blanching when the lesion is pressed is commonly found on vascular lesions. 

 

156: Which of the following is a fluorescent ectothrix dermatophyte? 
A. Trichophyton rubrum 
B. Trichophyton mentagrophytes 
C. Microsporum ferrugineum 
D. Trichophyton violaceum 
E. Microsporum nanum 

 

C. Fluorescent ectothrix dermatophytes include M. canis, M. audouinii, M. distortum, M. ferrugineum, and sometimes M. gypseum and T. schoenleinii ("Cats And Dogs Fight and Growl Sometimes.") Nonfluorescent ectothrix dermatophytes include T. mentagrophytes, T. rubrum, T. verrucosum, T. megninii, and M. nanum. Endothrix dermatophytes include T. rubrum, T. gourvilli, T. yaounde, T. tonsurans, T. schoenleinii, T. soudanense, and T. violaceum. 

 

157: Pernio or Chilblains is an inflammatory skin condition which is triggered by cold, wet, nonfreezing environmental conditions. Acral skin has violaceous discoloration accompanied by burning or itching. While avoidance and prevention is best, the most effective pharmacologic treatment is: 
A. Nifedipine 
B. Nicotinamide 
C. Phenoxybenzamine 
D. Psoralen+UVA 
E. Aspirin 

 

Nifedipine is effective in about 70% of patients with pernio in prevention of the development of new skin lesions. The other options listed, other than aspirin, are anecdotally suggested to be helpful.  

 

158: Which one of the following is responsible for maintaining a barrier to water loss in the stratum corneum? 
A. Involucrin 
B. Filaggrin 
C. Loricrin 
D. Transglutaminase 
E. Odland bodies 

 

Odland bodies, also known as lamellar granules, keratinosomes, and membrane-coating granules, are small organelles that are discharged from granular cells into the intracellular space of the granular layer of the epidermis. These bodies have two known functions: they mediate stratum corneum cell cohesion and they form a barrier to water loss. Odland bodies are round to oval, measure approximately 300 to 500 nm in diameter, and possess a trilaminar membrane and a laminated interior. They contain neutral sugars linked to lipids and/or proteins, hydrolytic enzymes, and free sterols.  

 

159: The main cause of nutritional disease in developed nations is: 
A. Unusual diets 
B. Inflammatory bowel disease 
C. Malabsorption syndromes 
D. Alcoholism 
E. Psychiatric illness  

 

Alcoholism is the main cause of nutritional disease in developed nations. Other conditions that cause nutritional disease include: unusual diets, postoperative state, psychiatric illness, inflammatory bowel disease, cystic fibrosis, surgical bowel dysfunction, and inborn errors of metabolism  

 

160: Which of the following HLA types is associated with early onset psoriasis? 
A. HLA-B13 or HLA-B17 
B. HLA-B27 
C. HLA-B57, HLA-Cw6, or HLA-DR7 
D. HLA-Cw2 
E. HLA-DR3 

 

HLA-B57, HLA-Cw6, or HLA-DR7 are most commonly associated with early onset type I psoriasis. The presence of HLA-B13 or B17 is associated with a 5-fold risk of developing psoriasis and are increase in guttate and erythrodermic psoriasis. HLA-B27 may be seen in pustular psoriasis. HLA-Cw2 is seen with late onset psoriasis, or type II. HLA-DR3 is commonly found with subacute cutaneous lupus  

 

161: A patient has on exam multiple thick banded livedo patterns on the shins with painful, stellate, necrotic and purpuric plaques on the shin. She is diagnosed with calciphylaxis. All the following are risk for this disease except: 
A. Dialysis 
B. Warfarin therapy 
C. Iron and vitamin D therapy 
D. Liver disease 
E. Smoking 

 

E. Most patients with calciphylaxis have chronic renal insufficiency and hyperparathyroidism. The associated risk factors are diabetes mellitus, obestiy, dialysis, warfarin therapy, iron and vitamin D replacement, weight loss, and liver disease 

 

162: Which sebaceous gland is located on the areola?
A. Montgomery's tubercles 
B. Tyson′s glands 
C. Zeis glands 
D. Meibomian glands 
E. Fordyce's spots 

 

Montgomery's tubercles are sebaceous glands not associated with hair follicles present on the areola. Tyson's glands on the labia minora and glans, meibomina and Zeis glands are present on the eyelids , and Fordyce's spots are located on the buccal mucosa. 

 

163: Jessner's solution includes: 
A. Lactic acid, alcohol, glycolic acid, resorcinol 
B. Lactic acid, alcohol, glycolic acid, TCA 
C. Lactic acid, alcohol, salicylic acid, glycolic acid 
D. Lactic acid, resorcinol, salicylic acid, glycolic acid 
E. Lactic acid, alcohol, salicylic acid, resorcinol 

 

E. Jessner's is a superficial peeling agent, which may be used to treat dyschromia and texture abnormalities. TCA and glycolic acid are not ingredients in this formulation 

 

164: Which antibody is associated with a positive prognosis in dermatomyositis? 
A. Anti-Jo 
B. Anti-Smith 
C. SRP 
D. Anti-Mi 
E. Aldolase 

 

Several antibodies have prognostic importance in dermatomyositis. Anti-Mi is associated with an improved prognosis. Anti-Jo is associated with a poor prognosis and lung involvement. SRP is also associated with a poor prognosis and heart involvement. 

 

165: Which of the following is NOT used for histologic examination for fungal infection? 
A. Chlorazol Black-E 
B. Gormori Methanamine Silver 
C. Periodic Acid Schiff 
D. Fontana-Masson 
E. Mayer's mucicarmine 

 

Chlorazol Black-E is a stain specific for chitin and is used in direct examination, not histologic preparations. The remaining options are histology stains useful in fungal identification in tissue. 

 

166: Which presentation of psoriasis is more common in children:
A. Acrodermatitis continua of Hallopeau
B. Guttate psoriasis
C. Keratoderma blennorragica
D. Pustular psoriasis
E. Erythrodermic psoriasis 

 

Guttate” stems from the Latin word for “drop.” It’s the second most common form of psoriasis. Psoriasis is an inflammatory skin condition that causes skin redness and irritation. It typically affects children and adults under 30 years old. Respiratoryillnesses or viral infections are common triggers. According to the National Psoriasis Foundation (NPF), about 10 percent of people who have psoriasis will develop this type of psoriasis. 

 

167: The protein component of primary cutaneous amyloidosis is: 
A. SAA protein 
B. AL protein 
C. Keratin 
D. Collagen 
E. Bp180 

 

Primary cutaneous amyloidosis presents as either macular or lichen amyloidosis. The protein component is keratin. Macular amyloidosis often presents over the upper back, while lichen amyloid presents over the shins. 

 

168: A 18 yo man presents for evaluation of foot lesions. There are thick hyperkeratotic plaques symmetrically on only the weight bearing plantar surfaces. What test(s) should this patient be referred for?
A. Thoracic CT
B. Head CT
C. Endoscopy
D. Hepatic ultrasound
E. Knee films 

 

The patient likley has Howel-Evans Syndrome. These patients present with symmetric focal weight bearing PPK in the second decade to adulthood. After the third decade, esophageal carcinoma can occur. These patients should have periodic endoscopic evaluation 

 

168: Which of the following are true for the sign of Leser-Trelat? 
A. Lesions are commonly located on the chest and back 
B. Classically described to appear in a “Christmas tree” pattern 
C. Usually associated with internal malignancies
D. Eruptive nature 
E. All of these answers are correct 

 

E Characterized by the sudden appearance of multiple SKs, the sign of Leser-Trelat may have lesions located anywhere in the body. However, the “eruption” is classically described to appear in a “Christmas tree” pattern commonly located on the chest and back. It may be associated with internal malignancies such as adenocarcinomas 

 

169: Once a keratinocyte leaves the basal cell layer, the normal transit time to stratum corneum is at least: 
A. 7 days 
B. 14 days 
C. 21 days 
D. 28 days 
E. 35 days 

 

Correct Answer: B Once a basal cell leaves the basal layer in humans, normal transit time to stratum corneum is at least 14 days. Transit time through the stratum corneum to desquamation requires 14 more days. 

 

170; Which of the following immunosuppressive agents may exert a protective effect against the development of skin cancers in organ transplant patients? 
A. Cyclosporine 
B. Rapamycin 
C. Tacrolimus 
D. Corticosteroids 
E. Azathioprine 

 

Rapamycin (also called sirolimus) is a macrolide antibiotic and a structural analog of FK 506. It is a potent immunosuppressive agent which inhibits mTOR (a member of P13K family kinases). Despite its immunosuppressive effects, preliminary data show a decreased incidence of skin cancer in organ transplant patients treated with rapamycin and postulate that it may exert a protective effect against cutaneous malignancies. 

 

171: A thick scale with matted hair is seen on a patients head. The most likely diagnosis is: 
A. Pityriasis amiantacea 
B. Traction alopecia 
C. Triangular alopecia 
D. Loose anagen syndrome 
E. Telogen effluvium 

 

Pityriasis amiantacea is a condition seen in pediatric patients that improves with age. There is thick scale, matted hair that may mimic seborrheic dermatitis or psoriasis. Treatment is keratolytics. 

 

172: Hebra nose deformity is characteristic of infection with which organism?
A. Serratia marcesens
B. Haemophilus influenzae
C. Streptococcus pyogenes
D. E. coli
E. Klebsiella pneumonia 

 

Hebra nose deformity is seen in Rhinoscleroma which is caused by Klebsiella pneumonia 

 

173: The most likely etiology of Jacquet’s diaper dermatitis is:
A. Multifactorial 
B. Candida
C. Trichophyton rubrum
D. Herpes simplex virus, Type 2
E. Group A beta-hemolytic streptococcus 

 

Jacquet’s diaper dermatitis is a multifactorial process. Yeast, irritants and moisture all contribute to the occurrence of this eruption 

 

174: What feature best distinguishes lichen planus from lichenoid drug eruption?
A. segmental hypergranulosis
B. parakeratosis
C. eosinophils
D. spongiosis
E. saw tooth pattern 

 

B. Lichenoid drug reaction and lichen planus are best distinguished clinically. However, there are clues to lichenoid drug, including parakeratosis and eosinophils. The presence of parakeratosis has been shown to be more sensitive than eosinophils in the diagnosis of lichenoid drug reaction 

 

175: Which of the following medications would be safe for use in pregnancy?
A. Tazarotene
B. Finasteride
C. Etretinate
D. Azeleic acid
E. 5-fluorouracil topically 

 

All of the listed medications except azeleic acid are pregnancy category X. Azeleic acid is a pregnancy category B medication.

 

 

176: The vector for relapsing fever that causes paroxysmal fevers, headaches, mylagias and erythematous or petechia macules in refugees is:
A. Pediculus humanus
B. Ornithodoros
C. Ixodes dammini
D. Ixodes pacificus
E. Ixodes ricinus    

 

Relapsing fever is caused by the human body louse Pediculus humanus and the borrelia species is B. recurrentis. The clinical features are paroxysmal fevers, headache, lymphocytoma,mylagias, erythematous or petechial macules on trunk and extremities.Louse-borne relapsing fever is transmitted by the human body louse and is generally restricted to refugee settings in developing regions of the world   

 

177: A 20-year-old patient presents with white patches of skin that have developed around his mouth and eyes. He has a past history of diabetes and you note that where a patch crosses the left eyebrow the hair is white. You suspect a diagnosis of vitiligo but also consider other causes of acquired hypopigmentation. Which of the following would NOT be in your differential diagnosis:
A Pinta
B Leprosy
C Piebaldism
D Pityriasis alba
E Post-inflammatory from lupus  

 

Piebaldism is a rare, autosomal-dominant syndrome with variable phenotype, presenting at birth. The characteristic clinical features are a white forelock and patchy absence of skin pigment . The depigmented lesions are static and characteristically occur on the anterior and posterior trunk, mid-upper arm to wrist, mid-thigh to mid-calf, and shins. 

178: Which of the following is most likely to be associated with alopecia areata?
A. Hypothyroidism
B. Seborrheic dermatitis
C. Tinea capitis
D. Congestive heart failure
E. Chronic kidney disease 

 

Alopecia area is associated with autoimmune diseases, especially thyroid disease, hypo- or hyperthyroidism. It presents with well-defined patches of alopecia without scale or erythema. 

 

179: The use of topical vitamin K has been shown to: 
A. Decrease the appearance of infraorbital pigmentation 
B. Minimize the appearance of telangiectasia 
C. Reduce the severity of laser-induced purpura 
D. Increase collagen production 
E. Induce keratinocyte differentiation 

 

 

Treatment of benign vascular lesions with the pulsed dye laser often produces significant postoperative purpura. Topical vitamin K has been shown to decrease the severity of laserinduced purpura although its mechanism of action has yet to be determined. No other cosmetic effects of topical vitamin K have been proven to be statistically significant. 

 

180:  The first cell type to migrate into a new wound in great numbers is the: 
A. Neutrophil 
B. Monocyte 
C. Macrophage 
D. Lymphocyte 
E. Mast cell 

 

Neutrophils migrate with monocytes concurrently, but arrive first in great numbers because of their abundance in circulation. Chemoattractants for the PMNs are fibrinogen, fibrin split products, C5a and leukotrienes. If wound contamination is controlled, PMN migration ceases within a few days and they become entrapped within the wound clot, undergo apoptosis or are phagocytosed by macrophages 

 

181: What is the most common subtype of molluscum contagiosum virus infection seen in HIV patients? 
A. MCV-1
B. MCV-2 
C. MCV-3 
D. MCV-4 
E. MCV-5 

 

B. The most common subtype of molluscum contagiosum virus is MCV-1. In HIV patients, however, MCV-2 is more common.  

 

182: Direct immunofluorescence is of NOT value in the diagnosis of:
A. SLE 
B. Lichen planus
C. Mixed connective tissue disease
D. Neonatal LE
E. Erythema multiforme 

 

D. DIF is of no value in the diagnosis of scleroderma, morphea, and neonatal LE. DIF of lichen planus is positive in the vast majority, with granular DEJ deposition and IgM and fibrinogen staining within cytoid bodies in the superficial dermis. In MCTD DIF shows IgG deposits within epidermal cell nuclei, and rarely along the DEJ. In SLE, while serology is more reliable, DIF can show DEJ deposition in the lupus band test. DIF of erythema multiforme shows immunoglobulin within superficial vessel walls, DEJ, and cytoid bodies 

 

183: Women with unwanted hair have the option of using a cream to help in the reduction of facial hair. This cream is:
A. Eflornithin
B. Spironolactone
C. Finasteride
D. Hydroxy-progesterone
E. Androstenedione 

 

 Women that have unwanted facial hair can use eflornithine and is also known as Vaniqa. It is applied to the skin for the reduction of unwanted facial hair. It inhibits ornithine decarboxylase located in the root of the hair follicle.  

 

 

184: Which skin complaints often worsens during pregnancy:
A Eczema
B Rosacea
C Plaque psoriasis
D Pityriasis rubra pilaris
E Lipomatosis 

 

B. A number of skin complaints tend to worsen during pregnancy, these include acne, rosacea, candidiasis and melasma. Some other skin conditions tend to improve in pregnancy such as eczema and psoriasis  

 

185: Melanocytes are derived from: 
A. Bone marrow 
B. Neural crest 
C. Mesodermal precursors 
D. Endodermal precursors 
E. Yolk sac derived 

 

Melanocytes are derived from neural crest precursors and migrate to the epidermis, hair matrix, retinal pigment epithelium, ear (stria vascularis), leptomeninges, and mucous membranes. The other options listed are incorrect and are not involved with melanocytes.  

 

186: What is the most common presentation of mucous membrane involvement of EBV infection?
A. Buccal ulcers with superficial slough
B. Erosions of the gingivae
C. Plication of the tongue
D. Palatal petechiae
E. Oral hairy leukoplakia  

 

D. The most common presentation of intra-oral involvement of EBV infection is pinpoint petechiae of the junction of the hard and soft palate. 

 

187: An 75 year old female with chronic lymphocytic leukemia develops disseminated varicellazoster infection. She is hospitalized for treatment. Rapid intravenous infusion of acyclovir has been associated with what complication?
A. Reversible obstructive nephropathy
B. Disseminated intravascular coagulation
C. Thrombocytopenia
D. Pulmonary fibrosis
E. Serum sickness 

 

Acyclovir is a guanosine analog. It is preferentially phosphorylated by viral thymidine kinase. It inhibits viral DNA polymerase, halting viral DNA synthesis by chain termination. Rapid intravenous infusion of acyclovir has been associated with a reverisble obstructive nephropathy 

188.  A patient with known chronic Hepatitis C virus infection presents with hyperkeratotic plaques of the bilateral feet. Biopsy reveals psoriasiform changes with superficial epidermal necrosis. Diet supplementation with what mineral has been shown to improve the appearance and symptoms of this cutaneous finding?
A. Zinc
B. Biotin
C. Magnesium
D. Calcium
E. Chromium  

 

Necrolytic acral erythema is characterized by hyperkeratotic well-defined plaques on the lower extremities, seen occasionally in patients with hepatitis C virus infection. Treatment consists primarily of treating the underlying infection. Diet supplementation with zinc has been shown to improve necrolytic acral erythema, even in the presence of normal serum zinc levels.  

 

189: The most sensitive microscopic test for fungal infection is: 
A. Potassium Hydroxide 
B. Potassium Hydroxide with DMSO 
C. Chlorazol Black E 
D. Calcofluor white 
E. Swartz Lamkins stain 

 

Calcofluor white is the most sensitive microscopic test for fungal infection. It is a glucan specific immunofluorescent stain. The remaining options are useful in direct microscopic examination, but not the most sensitive 

 

\190: Which of the following is an example of a delayed hypersensitivity reaction? 
A. Allergic contact dermatitis 
B. Anaphylaxis 
C. Latex allergy 
D. Transfusion reaction 
E. Serum sickness 

 

There are 4 types of Hypersensitivities: ("ACID") Type I: Anaphylactic and Atopic: Examples - Urticaria, Asthma, and Allergic Rhinitis Type II: cytotoxic: examples - Transfusion reactions, ABO incompatibility, Rh disease (erythroblastosis fetalis), Autoimmune reactions, Hemolytic disease of newborn, Goodpasture's syndrome Type III: Immune complex, Serum sickness, and arthrus reactions: examples -PAN, glomerulonephritis, SLE, Rheumatoid arthritis, and serum sickness Type IV: Delayed (cell-mediated)typesL examples -TB skin test, transplant rejection, contact dermatitis, interactions and skin repsonsiveness to bacteria, fungi, viruses, and protozoa, photo-allergies, insect bites, etc  

 

191: Subacute cutaneous lupus erythematosus has been associated with the ingestion of which of the following drugs? 
A. Phenytoin 
B. Allopurinol 
C. Terbinafine 
D. Trimethoprim/sulfamethoxazole 
E. Auranofin 

 

SCLE has been reported to be associated with terbinafine. This condition is often associated with anti-Ro (SS-A) and anti-La (SS-B) antibodies 

 

192: Which of the following systemic agents has been shown to be the most effective in the treatment of toenail onychomycosis?
A. Itraconazole
B. Griseofulvin
C. Ketoconazole
D. Terbinafine
E. Fluconazole 

 

D. Craford et al. reviewed the available literature examining the efficacy of systemic anti-fungals and performed a meta-analysis. Pooled analysis of cure rates at 11 and 12 months suggested that terbinafine was more effective than itraconazole. 

 

193: A 3-year-old girl is taken to her doctor due to a rash on the right upper arm. On examination multiple raised lesions of about 2 mm in diameter are seen. On close inspection a central dimple is present in the majority of lesions. What is the likely diagnosis?
A. Roseola infantum
B. Molluscum contagiosum
C. Kawasaki disease
D. Viral warts
E. Pityriasis rosea 

 

Molluscum contagiosum is caused by a pox DNA virus infection. It is typically seen in younger children and results in characteristic small, pearly, umbilicated lesions.  

 

194: Which of the following is the most common photodermatosis? 
A. Hydroa vacciniforme 
B. Chronic actinic dermatitis 
C. Actinic prurigo 
D. Polymorphous light eruption 
E. Solar urticaria 

 

Polymorphous light eruption is the most common photodermatosis. It is a idiopathic disease that usually appears in the first three decades of life and is more common in fair-skinned females. The pathogenesis is unclear, but is believed to be related to a type IV hypersensitivity reaction. Most lesions are erythematous pruritic papules, with the plaque form being less common. Lesions appear symetrically on exposed areas after a delay of several hours to several days. Patients with mild disease are treated with sun avoidance and a broad spectrum sunscreen. In more severe cases, hardening and desensitization can be accomplished or antimalarials can be used for resistant cases. For brief, sunny vacations, a short course of prednisone can be helpful. The other options are less common forms of idiopathic photosensitivity disorders. 

 

195: Which one of the following is not caused by a Candida infection? 
A. Angular cheilitis 
B. Balanitis 
C. Median rhomboid glossitis 
D. Leukoplakia 
E. Mycetoma 

 

E. Candidal infections may clinically present as thrush/leukoplakia, perleche/angular cheilitis, vulvovaginitis, balanitis, paronychia, onychomcosis, intertrigo, and folliculititis. Median rhomboid glossitis is now also associated with candidal infections. Mycetomas are caused by true fungi (eumycetoma) or filamentous bacteria (actinmycetoma). 

 

196: This medication is a source of anagen effluvium and can cause abrupt shedding of hair: 
A. Colchincine 
B. Hydrochlorothiazide 
C. Beta blockers 
D. Calcium channel blockers 
E. Procainamide 

 

Colchichine, mercury intoxication, thallium poisoning and severe protein deficiency can cause anagen effluvium. Hairs are usually broken and not shed 

 

197: Sebaceous glands secrete sebum through which of the following secretory mechanisms?
A. Holocrine and Merocrine
B. Merocrine
C. Holocrine and Apocrine
D. Holocrine
E. Apocrine 

 

Sebaceous glands exhibit holocrine secretion, whereby the sebocytes disintegrate in transit to the gland center, releasing their sebum contents. Merocrine secretion refers to the formation of intracellular secretory vesicles that translocate to the apical cell surface for secretion. Apocrine secretion refers to the process whereby secretory contents are packaged using the apical cell membrane, and 'pinched off' to achieve secretion. 

 

198: This type of psoriasis is associated with more severe body psoriasis. The location of this type is: 
A. Centrofacial psoriasis 
B. Palmoplantar psoriasis 
C. Scalp psoriasis 
D. Nail psoriasis 
E. Psoriatic arthritis 

 

Centrofacial psoriasis may be associated with more severe body psoriasis. Palmoplantar psoriasis can be difficult to treat. Patients that have psoriatic arthritis can be candidates for biologic therapy. 

 

199: What is the most common cause of erythroderma in non-HIV patients? 
A. Drugs 
B. Underlying malignancy 
C. Pre-existing dermatoses 
D. Infection 
E. Idiopathic 

 

Pre-existing dermatoses is most common cause in non-HIV patients, including atopic dermatitis, psoriasis, seborrheic dermatitis, chronic actinic dermatitis, mycosis fungoides, pityriasis rubra pilaris, and allergic contact dermatitis. Drugs are the second most common overall, and the most common in HIV patients . 

 

200: 16 year-old patient was recently diagnosed with HIV, the diagnosis is: 
A. Distal Onychomycosis 
B. Proximal Subungual Onychomycosis 
C. Proximal White Subungual Onychomycosis 
D. White Superficial Onychomycosis 
E. Paronychia with Candida Onychomycosis 

 

Proximal White Subungual onychomycosis is an AIDS marker, nails have a characteristic white opaque appearance beginning in the region of the lunula and extending distally under the nail plate. 

 

201; A patient has desquamative lesions around the mouth and is diagnosed with antiepiligrin cicatricial pemphigoid. The autoantibodies are: 
A. Anti-L-332 IgG autoantibodies 
B. Anti-32 autoantibodies 
C. Anti-IgM autoantibodies 
D. Anti-332 autoantibodies 
E. Anti-45 IgG autoantibodies 

 

Cicatricial pemphigoid affects the mucous membrane and genital areas. Anti-L-332 IgG autoantibodies are very specific for antiepiligrin cicatricial pemphigoid. 

 

202; Which of the following is a criterion for the diagnosis of Behcet'sdisease? 
A. Inflammatory bowel disease 
B. Uveitis 
C. Conjunctivitis 
D. Nasal septal perforation 
E. Lobular panniculitis 

 

Behcet's is a triad that includes oral ulcerations, genital ulcerations and uveitis. Behcet's also displays arthritis and gastrointestinal disease. Blindness is the most-feared outcome, and relates to the uveitis. 

 

203: Which of the following diseases does NOT respond with a Th1-type responses? 
A. Leishmaniasis which self-resolves 
B. Lepromatous Leprosy 
C. Tuberculoid Leprosy 
D. Allergic contact dermatitis 
E. Psoriasis 

 

Lepromatous leprosy is Th2 biased. The remaining conditions are Th1 predominant responses. Leishmaniasis which show strong cell-mediated immunity to the parasite and with self-resolving lesions are Th1 responses. In indolent/progressive leishmaniasis, a Th2 pattern is predominant 

 

204: Sentinel lymph node biopsy in malignant melanoma: 
A. Has gained acceptance for the treatment of MM of intermediate thickness (14mm) 
B. Is mainly being used for a more accurate staging 
C. Is not recommended 
D. None of these answers are correct 
E. Has gained acceptance for the work up of MM of intermediate thickness (14mm) and is mainly being used for a more accurate staging 

 

E. Sentinel lymph node biopsy has gained acceptance as part of the work up for melanomas 1-4 mm in thickness. Even though it has been widely accepted due to its low morbidity and high feasibility, the role of SLN biopsy in survival has not been established. It is mainly being used for a more accurate staging, prognosis and to determine if further adjuvant therapy is necessary. 

 

205: 35 years old woman is being treated for corticosteroid-induced rosacea with topical metronidazole and an oral tetracycline. She is very concerned about the redness of her face. Judicious use of what color concealer can reduce the appearance of redness on the skin?
A. Yellow
B. Lavender
C. Pink
D. Bronze
E. Green 

 

Green and red are on opposite sides of the color wheel and thus can “cancel” each other out. Green concealer can neutralize redness on the skin. 

 

206: Which of the following is not a feature of Cronkhite-Canada syndrome? 
A. Diarrhea 
B. Alopecia 
C. Lung carcinoma 
D. Dystrophic nails 
E. Hyperpigmented macules 

 

C Cronkhite-Canada syndrome is a rare, non-familial disease characterized by patchy alopecia, nail changes or loss, lentigines, inflammatory polyps, abdominal pain, and a protein losing enteropathy. 

 

207: A patient has multiple erythematous/violaceous psoriasiform dermatitis affecting the ears, nose, hands and feet. The patient is diagnosed with Bazex syndrome. The most appropriate course of action is to: 
A. Do an age appropriate cancer screening. 
B. Use topical corticosteroids 
C. To do a CT scan of the abdomen 
D. To order lipase and amylase levels 
E. To do a CT of the chest 

 

A patient with Bazex syndrome also known as Acrokeratosis Paraneoplastica should have an age appropriate cancer screening. Patients can also have nail dystrophy , acquired keratoderma, and the skin findings usually precede the underlying malignancy. 

 

208: H2 immune responses: 
A. Are associated with cell-mediated immunity 
B. Produce IL-6 
C. Produce IFN-gamma 
D. Produce TNF-beta 
E. Produce IL-2 

 

B TH1 cells produce IL-2, IFN-gamma, and TNF-beta, and are associated with cell-mediated immunity. TH2 cells produce IL-4, IL-5, IL-6, IL-10, and IL-13, and are associated with antibody-mediated immune responses. 

 

209: This dermatophyte has a growth requirement for inositol and thiamine 
A. Trichophyton equinum 
B. Trichophyton violaceum 
C. Trichophyton concentricum 
D. Trichophyton tonsuran
E. Trichophyton verrucosum 

 

Trichophyton verrucosum is a zoophilic fungus that requires thiamine and sometimes inositol for growth. T. violaceum and tonsurans only have a partial requirement for thiamine. Trichophyton equinum requires niacin. (Horses are nice) 

 

210: Which of the following is NOT associated with Hepatitis C disease? 
A. Mixed cryoglobulinemia 
B. PCT 
C. Single-stranded RNA viridae 
D. Leukocytoclastic vasculitis 
E. Single-stranded DNA viridae 

 

E. Hepatitis C virus (HCV) is a single-stranded RNA virus that is a member of the flaviviridae family. Approximately 20-30% of patients develop symptoms with acute infection and 70% will progress to chronic disease. Porphyria cutanea tarda or PCT is associated with HCV in a substantial percentage of patients; in one study, antibodies to HCV were found in 82% of PCT patients. Up to 80% of mixed cryoglobulinemia (MC) cases are associated with HCV. The incidence of lichen planus in HCV patients varies from region to region (0.1-35%). The histopathology of MC lesions is leukocytoclastic vasculitis. Another relatively common association is polyarteritis nodosa (PAN) which is also related to hepatitis B infection. Finally, generalized pruritis is a common complaint of HCV patients. 

 

211: Which of the following is not true about UVB radiation? 
A. Responsible for sunburn 
B. More intense in the summer than winter months 
C. Peaks at noon 
D. Decreased with high wind velocity 
E. Virtually all blocked by car window glass  

 

D. : D The UVB band extends from 290 to 320 nm. The UVB spectrum is recognized as the primary cause of sunburn, skin cancer, and other harmful effects on human skin. The UVA band extends from 320 to 400 nm. This spectrum is further subdivided into UVA-2 (320 to 340 nm) and UVA1 (340 to 400 nm). The UVA spectrum is recognized as a cause of immediate and delayed tanning reaction of skin, and several other effects including photoaging, skin photosensitization, and immunosuppression. UVC radiation comprises wavelengths shorter than 290 nm (from 200 to 290 nm). Notably, window glass filters out ultraviolet wavelengths shorter than 320 nm, so both UVB and UVC are effectively filtered by car window glass. UVB radiation is more intense during summer months compared to winter months and peaks during midday hours. It has been postulated that physical factors such as high temperature, high humidity, and wind can all increase susceptibility to UV-induced carcinogenesis. 

 

212: The term to describe hair shafts that are abruptly thinned and break off at the skin surface seen in anagen effluvium is: 
A. Pohl-Pinkus 
B. Epilated hair 
C. Trichotillomania 
D. Traction alopecia 
E. Telogen effluvium 

 

Pohl-pinkus is used to describe the abruptly thinned hair shafts. This is seen in anagen effluvium. The shedding of anagen hairs are always abnormal. 

213: Drugs that are associated with the exacerbation of pemphigus foliaceus include : 
A. Captopril 
B. Metoprolol 
C. Fluconazole 
D. Calcium channel blockers 
E. Statins 

A Pemphigus foliaceus is a form of pemphigus with superficial blisters. The autoantibodies are against Dsg1. Medications associated with the exacerbation of pemphigus foliaceus is sulfhydryl groups such as captopril, penicillamine and piroxicam. Metroprolol can exacerbate psoriasis 

214; Which of the following is a progestin used in oral contraceptives with low intrinsic androgenic properties? 
A. Norgestrel 
B. Novicane 
C. Norgestimate 
D. Levonorgestrel 
E. Spironolactone 

Desogestril, norgestimate and gestodone are progestins with low intrinsic androgenic properties. 


215: Which of the following diseases is caused by a virus?

A. Boutonneuse spotted fever
B. Colorado tick fever
C. Q fever
D. Rocky Mountain spotted fever
E. Human monicytic ehrlichiosis

 

Rocky Mountain spotted fever is caused by Rickettsia rickettsii. Human monicytic ehrlichiosis is caused by Ehrlichia chaffeensis. Colorado tick fever is caused by Reoviiridae. Boutonneuse fever is caused by Rickettsia conorii. Q fever is caused by Coxiella burnetti 

216; The most common side effect of treatment with interferon-alpha is:

  1. Liver toxicity
    B. Flu-like symptoms
    C. Spastic diplegia
    D. Weight loss
    E. Nausea 

The most common side effect of treatment with interferon-alpha is flu-like symptoms of fever, chills, myalgias, headache and arthralgias. Prophylactic administration of non steroidal anti-inflammatory medications may alleviate some of these symptoms 


217: The epidermis is comprised of what type of cells?
A. Keratinocytes, Melanocytes, Neutrophils, Langerhan cells
B. Keratinocytes, Endothelial cells, Merkel cells, Goblet cells
C. Keratinocytes, Melanocytes, Merkel cells, Goblet cells
D. Keratinocytes, Endothelial cells, Merkel cells, Langerhan cells
E. Keratinocytes, Melanocytes, Merkel cells, Langerhan cells 

E. The adult epidermis is composed of three basic cell types: Keratinocytes, melanocytes, and Langerhans cells. An additional cell, the Merkel cell, can be found in the basal layer of the palms and soles, the oral and genital mucosa, the nail bed, and the follicular infundibul 

218; Red or blonde hair pigmentation primarily results from
A. the reduced activity of DOPA dehydroxylase.
B. the presence of eumelanin.
C. the presence of pheomelanin. 
D. the reduced activity of tyrosinase.
E. the absence of melanin. 

Hair color is determined by melanocytes. The melanocytic activity of follicular melanocytes is coupled to anagen – hair is only pigmented when it is growing. Pigment is produced in the matrix area of follicle, above the follicular papilla. Eumelanin isthe pigment of brown/black hairs, and pheomelanin is the pigment of red/blonde hairs. Intensity of color is proportional to the amount of pigment. The absence of pigment produces white hair, and markedly reduced pigment produces gray hair 

219: An infant presents with multiple congenital hemangomas in an generalized distribution. What is the most serious associated condition?
A. None of the answers are correct
B. All of the answers are correct
C. Obstructive jaundice
D. Congestive Heart Failure
E. Portal hypertension 

High output congestive heart failure can lead to death in these children. Obstructive jaundice and portal hypertension both occur, but are less likely to cause death. The hemangiomas will undergo spontaneous regression. 

220: What is the mean time-frame for development of Stewart-Treves Syndrome?
A. Less than 1 year 
B. 1 year 
C. 5 years 
D. 10 years 
E. >20 years 

Stewart-Treves syndrome refers to an angiosarcoma arising from chronic lymphedema. It typically occurs as a complication of long-lasting lymphedema of the arm after mastectomy and/or radiotherapy for breast cancer. It is a very late complication of disease, usually developing several years later. The mean time-frame for development of Stewart-Treves Syndrome in the literature is 23 years (4-40 years). 

221: Histopathologically, a swiss cheese type appearance is characteristic to which of the following? 
A. Silica 
B. Tattoos 
C. Liquid silicone 
D. Sea urchin spines 
E. Zirconium 

Parafinoma and silicone granuloma both have a swiss cheese type appearance in the dermis to subcutaneous fat. The remaining options typically appear sarcoidal 


222: What family of medications is associated with xerosis? 
A. Sulfonylureas 
B. Beta blockers 
C. Calcium channel blockers 
D. Loop diuretics 
E. Cholesterol lowering agents 

E Medications that alter the lipid composition of the epidermis and stratum corneum may impair the normal barrier function of the skin. Cholesterol lowering medications like HMG-CoA reductase inhibitors and niacin may cause xerosis through this mechanism.  

223: An internal hordeolum can be a painful and erythematous. It is an infection to the meibomian glands and is caused by: 
A. Staphylococcus aureus 
B. Actinomyces israelii 
C. Borrelia burgdorferi 
D. Mycobacterium tuberculosis 
E. Streptococcus pyogenes 

Internal hordeolum are secondary infection of the Meibomian gland in the tarsal plate. The infectious agent is Staphylococcus aureus in 90-95% of cases. 

 

224: What is the condition which is a diagnostic cutaneous manifestation of sarcoid? 
A. Discoid lupus 
B. Lupus pernio 
C. Lupus vulgaris 
D. Granulomatous rosacea 
E. Rhinoscleroma 

Sarcoidosis is a systemic disease charcterized by non-caseating granulomas. Organ involved include the skin, lung, liver and eyes. Lupus pernio manifests as indurated, red-brown, swollen plaques of the nose, lips, cheeks and ears. 

225: Which of the following stains with Ulex europeus agglutinin I? 
A. Smooth muscle 
B. Eccrine glands 
C. Endothelial cells 
D. Macrophages 
E. Melanocytes 

Ulex europeus agglutinin I is a stain which identifies endothelial cells, keratinocytes, angiosarcomes and Kaposi's sarcoma. 

226: Which of the following is the first symptom of ataxia telangiectasias?
A. Facial telangiectases
B. Conjunctival telangictases
C. Breast cancer
D. Hematologic malgignancy
E. Cerebellar ataxia 

 Ataxia Telangiectasia (Louis-Bar syndrome) is an autosomal recessive disorder usually caused by mutations in the ATM gene, which is a chromosomal strand break repair enzyme. Cerebellar ataxia is the first sign, followed by telangiectases of the conjunctiva and skin. Thymic hypoplasia predisposes to increased infections. There is increased sensitivity to ionizing radiation resulting in hematologic and solid tumors. Female carriers have increased risk of breast cancer 

227: A female patient with adult acne is placed on spironolactone for her acne vulgaris. As a physician, you warn her that there is a black box warning against patients with a : 
A. Personal history of breast cancer 
B. Personal history of thyroid cancer 
C. Personal history of bladder cancer 
D. Personal history of colon cancer 
E. Personal history of rectal cancer 

A Spironolactone is an antiandrogenic potassium sparing diuretic. It can be used for women with adult acne that is recalcitrant to treatment. Although it is not first line, it helps to block androgens that cause acne in the jawline. Spironolactone has a black box warning against patient with a personal history of breast cancer or family history of breast cance 

228: Caterpillar bodies are seen in: 
A. Dyskeratosis congenital 
B. Porphyria cutanea tarda 
C. Amyloidosis 
D. Lipoid proteinosis 
E. Mucocele 

Caterpillar bodies are eosinophilic, elongated, segmented bodies located within the roofs of blisters and are considered to represent a specific histopathologic feature of porphyric bullous eruptions, including porphyria cutanea tarda and erythropoietic protoporphyria 

229: An 81-year-old woman presents with a rapidly growing lesion on her right helix. On examination there is a 2 cm diameter, well defined, skin coloured nodule with a central keratin plug. You suspect a diagnosis of keratoacanthoma. What treatment option should you recommend: 
A Radiotherapy 
B Punch biopsy 
C Incisional biopsy 
D Excision biopsy 
E Curettage and cauter 

Keratoacanthomas are common skin tumours that occur on sun exposed sites. They occur suddenly and expand rapidly up to 2 cm in diameter over a few weeks. They clinically appear as a well defined nodule with a central keratin plug. After a number of weeks they spontaneously regress without sequelae. Pathologically and clinically they are similar in appearance to well differentiated SCCs with significant cytological atypia and a high mitotic rate. For a pathologist to confidently diagnose keratoacanthoma they need to examine the whole specimen to account for the overall architecture of the lesion. It is thought that up to 10% of lesions diagnosed as keratoacanthomas are actually well differentiated SCCs and many dermatologists feel that all keratoacanthomas should be treated as SCCs. the patient should undergo an excision biopsy with minimal surgical margins. 

230: A newborn has a nodule over his lumbar spine. Skin biopsy reveals a lipoma. The next appropriate step is:
A. Observation
B. Excision of the lesion
C. Genetic testing
D. Imaging study
E. Malignancy work up  

The skin can provide an important clue to the presence of an underlying neural tube defect, such as meningomyelocele and encephalocele. Cutaneous lesions along the midline of the spine should always prompt consideration of this possibility. Although, midline neural tube defects are uncommon, early recognition and diagnosis of a spinal dysraphism can have important implications for early surgical correction and minimizing loss of neurologic function. Clues to the diagnosis include a midline dimple, tuft of hair, lipoma, or vascular lesion. In these instances imaging studies (MRI, CT, ultrasound) should be promptly initiated. 

231: Which of the following is a feature of Neurofibromatosis type II?
A. Lisch nodules
B. Optic gliomas
C. Juvenile posterior subcapsular lenticular opacities 
D. Congenital hypertrophy of the retinal pigment epithelium
E. Lester iris  

Neurofibromatosis type II is an autosomal dominant disorder caused by mutations in schwannomin/merlin. Clinical features include cutaneous schwannomas and neurofibromas, bilateral vestibular schwannomas, and juvenile posterior subcapsular lenticular opacities. 

232:  The most definitive HLA association with psoriasis is: 
A. HLA-Cw6 
B. HLA-B27 
C. HLA-B13 
D. HLA-B17 
E. HLA-B37 

HLA-Cw6 is associated with a 9-15x greater risk for developing psoriasis. All of the other HLA antigens listed are associated with various types of psoriasis, but at with lesser strength of association. 

233: Which of the following hypoglycemics is the most common cause of photosensitivity? 
A. Insulin 
B. Sulfonylureas 
C. Metformin 
D. Thiazolidinediones (i.e. rosiglitazone) 
E. Piroxicam 

The sulfonylurea hypoglycemics for diabetes are the most common type of medication of this class of medications. Piroxicam is not a hypoglycemic agent, but a common NSAID cause of photoallergy. 

234: The most common location for a basal cell carcinoma is: 
A. Lower eyelid 
B. Forehead 
C. Ears 
D. Back 
E. Shoulders 

The most common location for a BCC is the lower eyelid. It is the most common epithelial tumor of the eyelid 

235: This organism produces an endothrix tinea capitis: 
A. Trichophyton mentagrophytes 
B. Microsporum gypseum 
C. Microsporum nanum 
D. Trichophyton verrucosum 
E. Trichophyton soudanense 

Only T. soudanense in this group of choices produces an endothrix type of tinea capitis. Endothrix do not fluoresce. Trichophyton mentagrophytes, Microsporum nanum and Trichophyton verrucosum produce nonfluorescent ectothrix tinea capitis. Microsporum gypseum may produce a fluorescent or nonfluorescent ectothrix tinea capitis. 

236: Individuals with which of the following syndromes characteristically present with photosensitivity, mental retardation, a "wizened" appearance, "bird-headed" facies, and "Mickey Mouse" ears? 
A. Tay Syndrome 
B. Rothmund-Thomson Syndrome 
C. Hutchinson-Gilford Progeria Syndrome 
D. Cockayne Syndrome 
E. Werner Syndrome 

Cockayne Syndrome is AR, caused by defective excision repair, cross-complementing group 8 gene (ERCC8). It presents with photosensitivity, mental retardation, and cachectic dwarfism. Patients have a characteristic "wizened" appearance, "bird-headed" facies, and "Mickey Mouse" ears. Cataracts, deafness, pigmentary retinopathy, dental caries, and skeletal, GU, and endocrine abnormalities may be seen 

237: The joint most frequently affected in acne fulminans is the: 
A. Elbow 
B. Intervertebral 
C. Distal interphalangeal joints of the hand 
D. Sternoclavicular 
E. Sacroiliac 

Acne fulminans may be associated with lytic bone changes which are indicative of a sterile osteomyelitis. The sternoclavicular and chest wall joints are most frequently affected. 

238: Rare diseases are rare, but rare disease patients are numerous"

Brooke-Spiegler syndrome is a condition characterized by multiple skin tumors that develop from structures associated with the skin, such as sweat glands and hair follicles. People with Brooke-Spiegler syndrome may develop several types of tumors, including growths called spiradenomas, trichoepitheliomas, and cylindromas. The tumors associated with Brooke-Spiegler syndrome are generally benign (noncancerous), but occasionally they may become malignant (cancerous). Individuals with Brooke-Spiegler syndrome are also at increased risk of developing tumors in tissues in other areas, particularly benign or malignant tumors of the salivary or parotid glands and basal cell carcinomas. Brooke-Spiegler syndrome is caused by mutations in the CYLD gene. Susceptibility to Brooke-Spiegler syndrome has an autosomal dominant pattern of inheritance, which means one copy of the altered gene in each cell increases the risk of developing this condition. However, a second, non-inherited mutation is required for development of skin appendage tumors in this disorder.

 

239; This antiviral medicine doesn't require phosphorylation and may be used for acyclovir recalcitrant disease: 
A. Foscarnet 
B. Gancyclovir 
C. Famciclovir 
D. Valacyclovir 
E. Penciclovi 

 

Foscarnet is a non-competitive inhibitor of viral DNA polymerase. It does not require phosphorylation and can be used for acyclovir recalcitrant disease 

 

240: All of the following are true regarding Henoch-Schonlein Purpura EXCEPT: 
A. It is an IgG mediated small vessel vasculitis 
B. Direct immunofluorescence of lesional and peri-lesional skin will demonstrate C3 and fibrin deposits in small vessel wall 
C. It is often preceded by an upper respiratory infection 
D. It may be complicated by intussusception
E. It is self-resolving 

 

Henoch-Schonlein Purpura (HSP) is an IgA mediated small vessel vasculitis. Clinically, patients present with palpable purpura of the lower extremities and buttocks, GI vasculitis, and glomerulonephritis. 

 

241: Which of the following histologic features would be most helpful in differentiating lichenoid drug eruption from lichen planus?
A. Squamatization of the basal layer 
B. Band-like infiltrate with “Saw-tooth” rete ridges 
C. Parakeratosis and eosinophils
D. Presence of pruritus 
E. Civatte bodies 

 

C

 

242; This patient also has poliosis and deafness. What is the associated ophthalmologic finding? 
A. Normal ophthalmologic exam 
B. Retinal detachment 
C. Lisch nodules 
D. Corneal opacities 
E. Granulomatous uveitis 

 

Vogt-Koyanagi-Haradi syndrome (VKHS). VKHS is marked by bilateral granulomatous uveitis, vitiligo, deafness, poliosis, and alopecia. This syndrome begins with a meningoencephalitic phase with fever, malaise, headaches, nausea, and vomiting. Varying degrees of neurological impairment may be present. Recovery is usual complete. The second phase is the ophthalmic-auditory phase during which uveitis appears rapidly and can results in blindness. Treatment with systemic steroids may prevent blindness. Lisch nodules are iris hamartomas which are seen in neurofibromatosis. Corneal opacities are seen in X-linked ichthyosis. Retinal detachment can occur as a result of head trauma, as in shaken-baby syndrome. 

 

243: 1 melanocyte has contact with: 
A. 6 keratinocytes 
B. 12 keratinocytes 
C. 18 keratinocytes 
D. 30 keratinocytes 
E. 36 keratinocytes 

 

One melanocyte and 36 keratinocytes make up one epidermal melanin unit. This melanocytes transfers pigment to these keratinocytes. 

 

244: Which of the following sutures is the first to be absorbed? 
A. Catgut 
B. Polyglycolic acid 
C. Polyglactin 910 
D. Polydioxanone 
E. Polypropylene 

 

Catgut has a variable rate of absorption but typically lasts about 7-14 days. Thus, it would be the first suture to be absorbed. Polyglactin 910 (Vicryl) and polyglycolic acid (Dexon) are both absorbed in about 90 days. Polydioxanone (PDS) is an absorbable monofilament which lasts approximately 180 days. Polypropylene (Prolene) is not an absorbable suture. 

 

245: What is the treatment of choice for porphyria cutanea tarda? 
A. Phlebotomy 
B. Antimalarials 
C. Erythropoietin 
D. Oral iron supplementation 
E. Naproxen 

 

Porphyria cutanea tarda (PCT) is caused by a deficiency in the enzyme uroporphyrinogen decarboxylase. Phlebotomy is the treatment of choice and may improve enzymatic activity by removing iron, an inhibitor of the enzyme. Antimalarials and erythropoietin are alternative therapies. Oral iron supplementation may worsen PCT. Naproxen is a common cause of pseudoporphyria. 

 

246: What is the most common internal cause of intractable pruritus? 
A. Hepatitis C 
B. Hypothyroidism 
C. Hyperthyroidism 
D. Chronic renal failure 
E. Internal malignancy 

 

Chronic renal failure is the most common internal systemic cause of pruritus. Up to 49% of patients with chronic renal failure have pruritus. Other systemic causes of pruritus include liver disease, hepatitis C, hypo and hyperthryoidism, iron deficiency anemia, polycyhtemia vera, Hodkin's lymphoma, leukemia, carcinoid, internal malignancy, AIDS, and internal parasites. 

 

247: Roseola infantum is caused by which virus? 
A. Coxsackie virus A16 
B. Human herpes virus 6 
C. Parvovirus B19 
D. Epstein-Barr virus 
E. Pox virus 

 

Roseola infantum may be cause be either human herpes virus types 6 or 7. It is the sixth of the traditional exanthems of childhood. A diffuse, maculopapular eruption is typically preceded by a prodrome of fever and either respiratory or gastrointestinal symptoms. The infection usually occurs in the spring and the fall and nearly all children are seropositive for the virus by the age of 4. 

 

248: Tissue contraction begins: 
A. At 3rd day of wound healing 
B. During the 2nd week of wound healing 
C. After the first month of wound healing 
D. After the 3rd month of wound healing 
E. After the 9th month of wound healing 

 

Wound healing tends to be a predictable process that begins initially with the inflammatory stage. During this stage, clot formation occurs. This is the initial step in wound healing. Platelets, neutrophils and macrophages all migrate to the wound and secrete many mediators of wound healing. Epithelialization then begins hours after injury. This is followed by granulation tissue formation (four days after injury) and angiogenesis (first week of repair). Wound contraction ensues during the second week of healing. 

 

249: Which antibody is specific for CREST syndrome? 
A. Anti-mitochondrial 
B. Anti-histone 
C. Anti-ds DNA 
D. Anti-nucleolar 
E. Anti-centromere 

The antinuclear antibody (ANA) pattern most specific for CREST is the anti-centromere pattern. The specificity rate is approximately 50-90% and carries a more favorable prognosis than Scl-70. The target protein for the anti-centromere pattern is the kinetochore. 

 

250: A young male patient presents with a painful ulcer with a ragged edge after a new sexual encounter. What is the treatment of choice? 
A. Penicillin 
B. Doxycycline 
C. Azithromycin 
D. Ciprofloxacin 
E. Cephalexin 

 

Ulcers in the genital area in sexually active patient generally fall into two groups: painful and painless. Painless ulcers include syphilllis, lymphogranuloma venereum, and granuloma inguinale. The primary cause of painful erosion or ulcer is H. ducreyi or herpes simplex. An ulcer with a ragged edge that is painful is characteristic of H. ducreyi. The treatment of choice is azithromycin. 

 

251: Telogen hairs are in the resting phase and last for: 
A. 3 months 
B. 1 month 
C. 2 months 
D. 4 months 
E. 5 months

 

telogen hairs are at the resting phase and make up 10-15% of hairs and last for 3 months. Anagen is the growing phase and last for 2-6 years and catagen hairs last for 2 weeks.  

 

252: Which association is incorrect? 
A. Epidermolysis bullosa acquisita : inflammatory bowel disease 
B. Dermatitis herpetiformis : small bowel lymphoma 
C. Paraneoplastic pemphigus : Castleman’s 
D. Herpes gestationis : menopause 
E. Porphyria cutanea tarda : hemochromatosis 

 

Herpes gestationis is an immunoblistering disease of pregnancy. Unlike pruritic urticarial papules and plaques of pregnancy, it recurs with successive pregnancies. It is associated with Grave's disease and choriocarcinoma 

 

253: Platelets release which of the following factors to promote new tissue growth? 
A. Neutrophil chemotactic factor 
B. IL-1 
C. ADP 
D. TGF-alpha 
E. FGF 

 

Platelets release PDGF, TGF-alpha and TGF-beta which promote new tissue growth. FGF and IL-1 are released from monocytes as growth factors (monocytes also release PDGF, TGF-alpha and TGF-beta). ADP is released from platelets, but is not functioning as a growth factor. Neutrophil chemotactic factor is released from mast cells and is an inflammatory mediator. 

 

255: Which of the following is most likely to be a primary cutaneous infection? 
A. North American Blastomycosis 
B. Histoplasmosis 
C. Paracoccidioidomycosis 
D. Cryptococcosis 
E. None 

 

North American Blastomycosis is a primary cutaneous infection. All other mycoses are primary pulmonary infections, which may disseminate to the skin. 

 

256: All of the following statements regarding Bowen's disease are true EXCEPT: 
A. Lesions arising on the lower limbs are more common in men than in women 
B. The basement membrane remains intact on histopathology 
C. 5% of patients with Bowen's disease develop invasive squamous cell carcinoma 
D. Chronic sun exposure is a risk factor for Bowen's disease 
E. Mucosal Bowen's disease may appear as a verrucous plaque 

 

The most common locations for BD include the head and neck regions and the extremities. BD arising on the lower limbs is frequently found in women, whereas lesions located on the ears and scalp are more common in men. The basement membrane remains intact on histopathology. 5% of patients develop invasive SCC. Risk factors include chronic sun exposure, immunosuppression, HPV, arsenic exposure, and ionizing radiation. Mucosal BD may appear as a verrucous or polypoid plaque, as an erythroplakic patch, or as a velvety red plaque  

 

257: How soon does epithelialization begin after a skin wound occurs? 
A. Minutes 
B. Hours 
C. 2 days 
D. 4 days 
E. 6 days 

 

Re-epithelialization begins hours after an injury occurs. Keratinocytes from residual epithelial structures leapfrog each other. One to two days after injury, cells at the wound margin proliferate and begin to migrate into the wound. 

258: Which of the following cytokines is secreted by adipocytes? 
A. IL-2 
B. Il-4 
C. IL-6 
D. IL-10 
E. IL-12 

 

Adipocytes secrete tumor necrosis factor (TNF)É‘ ±, interleukin-6 (IL-6), leptin, adiponectin, angiotensinogen, and resistin. Specifically it has been shown that IL-6 mRNA levels in human subjects are significantly greater in adipose tissue than in other tissues known to express IL-6. It is thought that circulating IL-6 levels may be higher in obese patients, contributing to the development and progression of vascular disease in this patient population. 

 

257: Granulation tissue primarily contains: 
A. Collagen I 
B. Collagen III 
C. Collagen IV 
D. Fibrin 
E. Collagen VII 

 

Granulation tissue begins to form four days after injury and is composed of new capillaries, macrophages, fibroblasts, blood vessels and primarily collagen type III. Formulation of granulation tissue is dependent on the presence of fibronectin and progresses through an orderly sequence of matrix deposition: fibronectin to collagen III then finally collagen I 

 

258: Which of the following nutritional deficiencies is associated with perifollicular hemorrhage? 
A. Vitamin C 
B. Niacin 
C. Vitamin B12 
D. Folate 
E. Zinc 

 

Vitamin C deficiency, or scurvy, may lead to perifollicular hemorrhage. Niacin deficiency may result in a photo-distributed eruption, diarrhea, and psychiatric symptoms (diarrhea, dermatitis, and dementia). Vitamin B12 may lead to glossitis with a shiny, painful tongue. Zinc deficiency may result in a periorificial dermatitis.  

 

259: Which of the following chemical peels does not need to be neutralized? 
A. Salicylic acid 
B. TCA 40% 
C. Glycolic acid 
D. Lactic acid 
E. Phenol 

 

Salicylic acid is a very superficial chemical peel that is used for acne, milia, keratolysis. This peel localizes to the pores given its lipophilic nature. It is a self-neutralizing peel. 

 

260:  Which form of sarcoidosis is associated with camptodactyly? 
A. Lofgren's syndrome 
B. Heerfordt's syndrome 
C. Darier-Roussy 
D. Mikulicz syndrome 
E. Blau syndrome 

 

Camptodactyly is a flexion contracture of the 3rd through the 5th proximal interphalangeal joints and elbows bilaterally. Blau syndrome is an autosomal dominant form of sarcodosis due to a defect in the CARD15 gene manifesting also with arthritis, cutaneous sarcoid, uveitis, and synovial cysts. It does not have lung or visceral involvement. 

 

261; A 32 year-old pregnant woman presents for treatment comedonal and moderately inflammatory acne. You discuss treatment options with her, and she asks about the evidence for safety in pregnancy of various treatments. According to the FDA classifications, which of the following treatments either shows no risk to the fetus in controlled studies (but may show risk to animals), or shows no risk in animal studies (but no human studies have been conducted)? 
A. Benzoyl peroxide 
B. Trimethoprim-sulfamethoxazole 
C. Topical tretinoin 
D. Azeleic acid 
E. Tetracycline 

 

The FDA classifies medication safety for use in pregnancy as follows: Category A: Controlled studies in humans show no risk to fetus. Category B: Controlled human studies show no risk (but may show risk to animals), or no risk in animal studies (but no human studies have been conducted). Category C: Risk to human fetus cannot be ruled out, studies are lacking; animal studies are equivocal. Category D: Controlled studies show risk, but in some instances benefits may outweigh risks. Category X: Contraindicated in pregnancy. The question describes category B. Azeleic acid is a category B medication. Benzoyl peroxide, topical tretinoin, and TMP/SMX are category C medications. Tetracycline is a category D medication 

 

262; Which of the following statements about electromagnetic radiation is MOST correct? 
A. Electromagnetic radiation can be conceptualized as packets of power called photons 
B. The energy of photons is proportional to the wavelength 
C. The energy of photons is inversely proportional to the frequency 
D. Longer wavelengths penetrate the skin more deeply 
E. Electromagnetic radiation is measured in watts 

 

Longer wavelengths penetrate the skin more deeply. Electromagnetic radiation can be conceptualized as packets of ENERGY called photons. The energy of photons is proportional to the FREQUENCY and inversely proportional to WAVELENGTH. Electromagnetic radiation is measured in wavelength 

 

 

263; Erythema gyratum repens is known to be associated with all of the following malignancies except: 
A. Lung carcinoma 
B. Breast carcinoma 
C. Cervical carcinoma 
D. Bladder carcinoma 
E. Bone carcinoma 

 

Erythema gyratum repens presents more commonly in men than in women, and appears clinically as concentric erythematous rings with trailing scale on the trunk and proximal extremities. The skin is described as having a "wood grain" appearance. Skin findings often precede the diagnosis of an associated malignancy. Lung carcinoma is the most commonly associated malignancy, but it has also been reported in association with breast, cervical, bowel, prostate, gastric/bowel and bladder cancer. 

264: Monotherapy for acne with topical antibiotics is discouraged because of: 
A. Slow onset of comedolytic action 
B. Potential for irritation 
C. Lack of anti-inflammatory action 
D. Potential for bacterial resistance 
E. Poor patient compliance 

 

opical antibiotics reduce the population of P. acnes on the skin, and thus are indirectly antiinflammatory. In contrast to topical retinoids, topical antibiotics are not comedolytic. They are generally well tolerated by patients. 

 

265; Which of the following eye findings is caused by the rupture of Bruch's membrane? 
A. Angioid streaks 
B. Blue sclerae 
C. Retinal detachment 
D. Ruptured globe 
E. Keratoconus 

 

The rupture of Bruch's membrane causes angioid streaks in pseudoxanthoma elasticum. Bruch's membrane is the innermost layer of choroid with a central layer of elastic fibers. The other findings are found in Ehlers-Danlos syndrome and are not related to Bruch's membrane. 

266: Which of the following vehicles is correctly defined 
A. Ointment-water in oil emulsion 
B. Gel-oil in water emulsion 
C. Cream-Semisolid emulsion in alcohol base 
D. Lotion/Solution-Cream in water 
E. Foam - powder in cream 

 

Various vehicles are used for different clinical situations. Ointments are water in oil emulsions. Creams are oil in water emulsion. Gels are semisolid emulsion in alcohol base . Lotions/Solutions are powder in water (some oil). Foams are liquid and/or solid materials in a gaseous medium. e. All are correct (false) 

 

267: The risk of developing systemic involvement in a patient with nodular amyloidosis is: 
A. 1-4% 
B. 5-10% 
C. 25-30% 
D. 50-60% 
E. >95%  

 

Nodular amyloidosis is a rare type of primary cutaneous amyloidosis made up of AL protein. The risk for systemic involvement is 7%. 

 

268; Diarrhea, Dementia and a photosensitive dermatitis are associated with a deficiency of which vitamin? 
A. Niacin 
B. Biotin 
C. Thiamine 
D. Riboflavin 
E. Pyridoxine 

 

Niacin (Vitamin B3) deficiency is associated with a photosensitive dermatitis, diarrhea and dementia. The photosensitive dermatitis classically involves the face, neck and upper chest (Casal's necklace) and forearms . Other findings may include angular cheilitis and thickening and hyperpigmentation of skin overlying bony prominences. 

 

269; Embryologically, epidermal stratification occurs at approximately what estimated gestational age? 
A. 4 weeks 
B. 8 weeks 
C. 12 weeks 
D. 16 weeks 
E. 20 weeks 

 

Epidermal stratification begins at about 8 weeks estimated gestational age and is completed by the 2nd trimester. Epideral stratification occurs when an 'intermediate layer' develops between the epidermal basal cells and the overlying layer of periderm cells. The periderm is an embryonic structure that covers the primitive epidermis until keratinization begins, at which point the periderm sloughs off and contributes to the protective covering of the newborn, the vernix caseosa. This intermediate layer is highly proliferative, such that by 24-25 weeks estimated gestaional age, the epidermis consists of 4 or 5 layers, in addition to the degenerating periderm. 

 

270; Patients with pemphigus vulgaris have lesions that start in the mouth in approximately : 
A. 70% of patients 
B. 50% of patients 
C. 30% of patients 
D. 20% of patients 
E. 10% of patients 

 

In 70% of patients, pemphigus vulgaris starts in the mouth and other mucosal surfaces can also be involved 

 

271; Which enzyme can be a useful adjunct test to diagnose zinc deficiency where the serum zinc level is normal or near normal? 
A. Zinc sulfatase 
B. Alkaline phosphatase 
C. AST 
D. ALT 
E. Creatinine kinase 

 

A low serum alkaline phosphatase, a zinc dependent enzyme, may be a valuable adjuctive test where the serum zinc level is normal or near normal. The diagnosis of zinc deficiency should be suspected in at-risk individuals with acral or periorificial dermatitis. It is usually confirmed by low serum zinc levels.  

 

272; Pseudoxanthoma elasticum is caused by mutations in which of the following genes? 
A. Fibrillin 1 
B. Elastin gene 
C. Lysyl oxidase 
D. ABCC6 gene 
E. Collagen 5 

 

Pseudoxanthoma elasticum is caused by mutations in the ABCC6 gene, which is an ATP-using cell transporter. Elastin and lysyl oxidase mutations cause cutis laxa, fibrillin 1 mutations cause Marfan syndrome, and collagen 5 mutations cause Ehlers-Danlos syndrome. 

 

274; Black nail is caused by: 
A. Trichophyton mentagrophytes 
B. Proteus mirabilis 
C. Pseudomonas spp. 
D. Staphylococcus aureus 
E. Hortaea werneckii  

 

B Nail infections with Proteus mirabilis. T. mentag causes white discoloration of the nail and Pseudomonas will cause a green or yellow/green discoloration. Hortaea werneckii causes tinea nigra. S. aureus does not cause black nails. 

 

275: Compared with the general population, what is the overall risk of developing cutaneous and systemic malignancies in organ transplant recipient? 
A. 4 fold 
B. 10 fold 
C. 25 fold 
D. 50 fold 
E. 100 fold 

 

Organ transplant recipients are at increased risk of having both systemic and cutaneous develop. The estimated increased risk is 3-4 times that of the general population. However, the risk of skin cancer alone is much higher (e.g. SCC “ 65x increase, BCC 10-fold increase, Melanoma 3.4x increase). This increased risk is thought to be partially due to the immunosuppressant agents used to prevent graft rejection. 

 

276; 

All of the following are reported cutaneous side effects of zidovudine EXCEPT: 
A. Trichomegaly 
B. Diffuse and oral hyperpigmented macules 
C. Hyperpigmented streaks in nails 
D. Periungual pyogenic granulomas 
E. None of the above (all are reported side effects  

 

E All of the following are cutaneous side effects reported with zidovudine, a nucleoside HIV reverse transcriptase inhibitor. Periungal/paronychial eruptions resulting in pyogenic granulomalike lesions have also been reported with other HIV medications, including indinavir and lamivudine 

 

277; What is the most common presentation of mucous membrane involvement of EBV infection? 
A. Buccal ulcers with superficial slough 
B. Erosions of the gingivae 
C. Plication of the tongue 
D. Palatal petechiae 
E. Oral hairy leukoplakia 

 

D The most common presentation of intra-oral involvement of EBV infection is pinpoint petechiae of the junction of the hard and soft palate. The associated eponym is Forchheimer spots. Forchheimer spots are not pathognomonic for EBV infection, as they can be seen in measles, rubella, and scarlet fever. While oral hairy leukoplakia is associated with EBV infection, it is not the most common mucous membrane sign 

278; This antihistamine is used for diarrhea in mastocytosis: 
A. Cromolyn 
B. Doxepin 
C. Cimetidine 
D. Ranitidine 
E. Hydroxyzine 

 

Cromolyn sodium blocks mast cell degranulation and is used for controlling diarrhea in mastocytosis. 

 

279; A obese 52-year-old woman presents with dark brown velvety plaques and acrochordons on the neck and axillae. Which of the following is most likely to be associated with her cutaneous findings? 
A. Diabetes mellitus 
B. Hypertension 
C. Hypercholesterolemia 
D. Hypertriglyceridemia 
E. Peripheral vascular disease 

 

This obese patient has acanthosis nigricans and acrochordons, both of which may be associated with insulin resistance and diabetes mellitus. While she may very well have hypertension, hyperlipidemia, hypercholesterolemia, and peripheral vascular disease, these are not significantly related to her cutaneous findings. Acanthosis nigricans may also be seen in patients with gastric adenocarcinoma. 

 

280; Dental enamel pits are seen in which of the following conditions?
A. Hypomelanosis of Ito
B. Tuberous sclerosis
C. Hyper-IgE syndrome
D. Letterer-Siwe disease
E. Jackson Sertoli syndrome 

 

Tuberous sclerosis is an autosomal dominant condition caused by mutations of the TSC1 (hamartin) or TSC2 (tuberin) genes. These are tumor suppressor genes. Skin findings include hypopigmented macules, connective tissue nevus, facial angiofibromas, periungual fibromas and cafe au lait macules. Dental enamel pits and gingival fibromas are oral findings that are associated with this condition. The remaining conditions do not have dental pits. 

 

281; What is the major component of the anchoring fibril?
A. Type III Collagen
B. Type I Collagen
C. Type VII Collagen
D. Type IV Collagen
E. Laminin 5 

 

Anchoring fibrils are made of Type VII collegen. Anchoring plaques are made of Type IV collagen, and interact with a network of Type I And Type III collagen fibers in the dermis 

 

282; The anti-HIV medication best known for causing a severe reaction which can result in fatality upon rechallenge is: 
A. Indinavir 
B. Didanosine 
C. Zidovudine 
D. Abacavir 
E. Nevirapine 

 

The hypersensivity reaction associated with abacavir usually resolves with cessation of the drug, however upon rechallenge the reaction can be life-threatening 

 

283; Which cytokine is NOT upregulated in atopic dermatitis patients? 
A. IL-13 
B. IL-4 
C. IL-5 
D. IL-10 
E. IFN-gamma 

 

E Interferon gamma is Th1 cytokine which downregulates Th2 responses. The remaining are Th2 cytokines active in atopic dermatitis. IL-4 is a B-cell growth factor and active in signaling isotope switching from IgM to IgE. IL-5 is an eosinophil growth factor. IL-10 downregulates Th1 immunity and IL-13 signals isotope switching along with IL-4. 

 

284; Patient presents with hemorrhagic onycholysis. The drug class most commonly associated with this finding is: 
A Quinolone antibiotics 
B Systemic retinoids 
C Calcineurin inhibitors 
D Taxanes 
E Tetracyclines 

 

Taxane probably cause nail changes more commonly than other drugs. Cutaneous toxicity has been reported with taxanes and includes erythema and desquamation, involving primarily the hands. Taxanes exert their cytotoxic effect by reversibly binding the B-subunit of tubulin, thereby inducing tubulin polymerization and inhibiting microtubule depolymerization. A balance between polymerization and depolymerization is needed for normal microtubule function. Taxanes disrupt this balance, leading to arrest at the G2/M phase of the cell cycle 

 

285; Pineapples contain which of the following proteolytic enzymes? 
A. None of these answers are correct 
B. All of these answers are correct 
C. Bromelin 
D. Phorbol esters 
E. Ranunculin 

 

Bromelin is a proteolytic enzyme found in pineapples. It is granted access to dermal vessels by calcium oxalate-induced fissures. Phorbol esters are found in plants of the Euphorbiaceae family. Ranunculin is an irritant found in buttercups (Ranunculaceae family) 

 

286; The keratoacanthoma variant characterized by the sudden appearance during childhood or adolescence of multiple KAs is called: 
A. Gorlin syndrome 
B. Xeroderma pigmentosa 
C. Ferguson-Smith 
D. Grzybowski 
E. None of these answers are correct 

 

The Ferguson-Smith is a keratoacanthoma variant characterized by the sudden appearance during childhood or adolescence of multiple KAs that may resolve and later-on reappear. This condition is inherited in an autosomal dominant pattern 

 

287; Which of the following biologic therapies is pregnancy category C? 
A. Etanercept 
B. Alefacept 
C. Infliximab 
D. Efalizumab 
E. Adalimumab 

 

All of the above drugs are pregnancy category B except efalizumab which is category C 

 

290; Side effects of PUVA include all of the following except: 
A. Headache 
B. Pruritus 
C. Neutropenia 
D. Insomnia 
E. Nausea 

 

Side effects of psoralen with ultraviolet A light (PUVA) include side effects which are due to phototoxic effects including pruritus, photoonycholysis, friction blisters, ankle edema and hypertrichosis. In addition, there are adverse effects from methoxypsoralen including gastrointestinal and neurologic effects, hepatotoxicity and exanthems. Neutropenia is not a described side effect of PUVA. 

 

291; What is the most common cause of nonpigmented fixed drug eruption? 
A. Naproxen 
B. Tetracycline 
C. Sulfonamides 
D. Phenolphthalein 
E. Pseudoephedrine 

 

Pigmented incontinence is usually prominent in a fixed drug eruption; yet occasionally, fixed drug reactions do not result in long-lasting hyperpigmentation. The so-called nonpigmented fixed drug eruption is distictive. Pseudoephedrine hydrochloride is by far the most common cause. 

 

292; The Gorlin syndrome is characterized by: 
A. Multiple BCCs during childhood 
B. Macrocephaly 
C. Odontogenic keratocysts of the jaw 
D. Autosomal-dominant inheritance pattern 
E. All of these answers are correct 

 

Gorlins syndrome is characterized by the appearance of multiple BCCs during childhood, odontogenic keratocysts of the jaw, and skeletal defects. It is inherited in an autosomal dominant pattern. 

 

293; A patient with a blue-red discoloration of the nail plate reports that the same finger becomes very tender when exposed to the cold. You suspect a:
A. Pyogenic granuloma
B. Glomus tumor
C. Periungual verruca
D. Mucous cyst
E. Pterygium 

 

Glomus tumors are tumors of the arterio-venous anastamosis of the digital dermis. They occur most frequently in the nail bed. The commonly have a bluish-red discoloration and may be tender or painful with exposure to heat or cold 

 

294; Keratinocytes are derived from which of the following:
A. Endoderm
B. Neural Crest
C. Bone marrow precursors
D. Ectoderm
E. Mesoderm 

 

As implied by the root 'ecto', a prefix meaning "outer", the keratinocytes of the epidermis are derived from the ectoderm. The other layers do contribute cell populations that are present in the skin. 

 

295; A specific marker of Merkel cells is
A. Envoplakin
B. Cytokeratin 10
C. Loricrin
D. Cytokeratin 20 
E. Cytokeratin 15 

 

Cytokeratin 20 is a specific marker for the Merkel cell. Merkel cells are mechanoreceptors located at body sites requiring high tactile sensitivity. Keratinocyte deformation results in a secretion of chemokines by Merkel cells, which make synaptic connection with neurons. 

 

296; Characteristic eosinophilic abscesses are seen in: 
A. Bullous drug 
B. Pemphigus vegetans 
C. Herpes gestationis 
D. Incontinentia pigmenti 
E. Paraneoplastic pemphigus 

 

Eosinophilic abscesses with minimal to no spongiosis in a hyperplastic epidermis are characteristic of pemphigus vegetans. 

 

297; The most common cause of fixed drug eruption is: 
A. Tetracyclines 
B. Macrolides 
C. Naproxin 
D. Pencillins 
E. Cephalosporins 

 

The most common cause of fixed drug eruption is tetracyclines. There is also a risk of brown discoloration of the teeth and delayed bone growth it should not be used in children in less than 9 years old. 

 

298; Which class of medication works by inhibiting DNA gyrase? 
A. Quinolones 
B. Tetracyclines 
C. Aminoglycosides 
D. Cephalosporins 
E. Carbapenem 

 

Penicillins, cephalosporins, carbapenems, and vancomycin work on the cell wall. Quinolones inhibit DNA gyrase. Tetracyclines and aminoglycosides work at the 30 S ribosomal subunit. 

 

299; Mee's lines are: 
A. Double white transverse lines from abnormal vascular bed 
B. Brownish macules beneath the nail plate 
C. Local or diffuse hyperkeratotic tissue that develops on the lateral or proximal nail folds 
D. Transverse white lines that affect all nails, grow out with nail growth 
E. Vertical black lines on a single or multiple nails 

 

Mee's lines are transverse white lines that affect all nails, grow out with nail growth. They are seen in arsenic poisoning, rheumatic fever, congenital heart failure, leprosy and with significant systemic disease. Brownish macules beneath the nailplate are oil spots, seen in psoriasis. Onychophosis is the local or diffuse hyperkeratotic tissue that develops on the lateral or proximal nail folds. The double white transverse lines from abnormal vascular bed are Muehrcke's lines which are caused by a nephrotic syndrome, low albumin, liver disease or malnutrition. 

 

300; The etiologic agent responsible for White Piedra is: 
A. Candida albicans 
B. Pityrosporum obiculare 
C. Corynebacterium tenuis 
D. Piedra hortai 
E. Trichosporon ovoides 

 

Patients with white piedra are infected with Trichophyton cutaneum also known as Trichophyton beigelii. The treatment should be oral azole antifungals and shampoos without shaving the scalp. 

 

301; All of the following statements regarding mast cells are true EXCEPT: 
A. Mast cells reside near small blood vessels 
B. Mast cells protect mucosal surfaces against pathogens 
C. Mast cells release substances that affect vascular permeability 
D. Mast cells have receptors for certain fragments of complement on their surface 
E. None of the above (all are true) 

 

Mast cells are the central cell in immediate-type hypersensitivity, and are very important in cutaneous inflammation. They reside near small blood vessels, and when activated release substances that affect vascular permeability. They orchestrate allergic responses and protect mucosal surfaces against pathogens. They have receptors for IgE and certain fragments of complement on their cell surface 

302. Kveim-Siltzback test is useful in the diagnosis of: 
A. Scarlet fever 
B. Sarcoidosis 
C. Leprosy 
D. Leishmaniasis 
E. Chancroid 

 

Kveim-Siltzback test is for sarcoidosis, The test is done by intradermal injection of a suspension from granuloma-containing spleen, lymph node, or other tissue from a confirmed case of sarcoidosis. A positive test is characterized by the formation of a papule at the site of injection within 4-6 weeks which, on microscopic examination, exhibits non-necrotizing granulomas and the absence of foreign material. This teast is rarely done nowdays because of the difficulties involved in preparation, standardization and validation of the test material as well as significant variation in the sensitivity and specificity of test suspensions obtained from different sources. 

 

303; In the epidermis, the cell most responsible for antigen detection and processing is the
A. Melanocyte
B. Merkel Cell
C. Langerhans cell 
D. CD4+ T cell
E. Keratinocytes 

 

The Langerhans cell is a bone narrow-derived, antigen-presenting cell found in all layers of the epidermis, oral mucosa, esophagus, and vagina. Langerhans cells ingest and process antigens, mature, migrate to a local lymph node, and then present the antigen to a na′ve (or resting) T cell, activating that T cell. The Langerhans cell is central to the pathogenesis of atopic dermatitis, psoriasis, allergic contact dermatitis, and certain infections, such as Leishmaniasis. 

 

304; Anagen effluvium is best described as
A. a cessation of mitotic activity in rapidly dividing hair matrix cells 
B. a scarring alopecia affecting only anagen stage follicles
C. a cessation of mitotic activity in resting hair matrix cells
D. an abrupt transition of telogen to anagen in resting hair matrix cells
E. an abrubt transition from anagen to catagen in rapidly dividing hair matrix cells 

 

Anagen effluvium results from an outside stimulus – most often an antimetabolite, chemotherapeutic drug – inducing an abrupt cessation of hair matrix cell mitotic activity. This process occurs within days to weeks of the stimulus, and is reversible with cessation of the drug therapy 

 

 

305; Tyrosinase positive albinism (oculocutaneous albinism type 2) is caused by a mutation in which of the following:
A. Tyrosinase
B. C-kit
C. NEMO
D. Tyrosinase related protein 1
E. P gene 

 

Oculocutaneous albinism (OCA) type 1 (Tyrosinase negative albinism) is caused by mutations in the tyrosinase gene. OCA type 2 (tyrosinase positive albinism) is caused by mutations in the P gene. OCA type 3 is caused by mutations in the tyrosinase related protein 1 gene. C-kit mutations cause piebaldism and NEMO mutations cause incontinentia pigmenti. 

 

306. The most likely etiology of Jacquet’s diaper dermatitis is:
A. Multifactorial 
B. Candida
C. Trichophyton rubrum
D. Herpes simplex virus, Type 2
E. Group A beta-hemolytic streptococcus  

 

Jacquet’s diaper dermatitis is a multifactorial process. Yeast, irritants and moisture all contribute to the occurrence of this eruption 

 

307; Which of the following is the vector for Dengue fever?
A. Lutzomyia verrucarum
B. Phlebotomus papatasii
C. Xenopsylla cheopis
D. Aedes aegypti
E. Culex mosquito 

 

Dengue fever also known as "break-bone" fever is characterized by sudden high fever, backache, retro-orbital pain, bone/joint pain, weakness, and malaise. It is caused by an arbovirus (RNA virus) which is transmitted by Aedes aegypti, a species of mosquito 

 

308; Which virus is most commonly associated with oral hairy leukoplakia?
A. Human papilloma virus
B. Epstein-Barr virus
C. Parvovirus
D. Pox virus
E. Herpes virus 

 

Oral hairy leukoplakia is an oral mucosal infection most often caused by the Epstein-Barr virus, which occurs in immunocompromised patients. Clinically, thick, white plaques are noted on the lateral sides of the tongue, often fissured 

 

309; Lipomembranous change is seen in:
A. Sclerosing panniculitis 
B. Sebaceous carcinoma
C. Cystic sebaceous adenoma
D. Hibernoma
E. Mucocele 

 

Lipomembranous change is a non-specific histologic pattern that is most commonly seen in lipodermatosclerosis, which is also known as sclerosing panniculitis; this condition may be secondary to venous stasis 

 

310; The major protein component of the cornified envelope is
A. Plectin
B. Envoplakin
C. Loricrin 
D. Transglutaminase
E. Desmoplakin 

 

Loricrin is the major component of the cornified envelope (CE). The proteins of the CE are synthesized in the spinous and granular layers. 

 

311; Sebaceous glands
A. respond to adrenergic neural activity, exclusively.
B. respond to the local release of cytokines from inflammatory cells.
C. respond to chemical stimuli such as hormones. 
D. respond to cholinergic neural activity, exclusively.
E. respond to both adrenergic and cholinergic stimuli. 

 

Sebaceous glands are androgen-responsive holocrine glands that enlarge at puberty. Meibomian glands of the eyelids are modified sebaceous glands. Sebaceous glands are found everywhere on the skin except the palms and soles. Fordyce’s condition involves free sebaceous glands on the vermillion border of the lips and on the buccal mucosa. Eccrine glands are thermoregulatory structures that respond to cholinergic stimulation. 

 

312; A 18 yo man presents for evaluation of foot lesions. There are thick hyperkeratotic plaques symmetrically on only the weight bearing plantar surfaces. What test(s) should this patient be referred for?
A. Thoracic CT
B. Head CT
C. Endoscopy
D. Hepatic ultrasound
E. Knee films 

 

The patient likley has Howel-Evans Syndrome. These patients present with symmetric focal weight bearing PPK in the second decade to adulthood. After the third decade, esophageal carcinoma can occur. These patients should have periodic endoscopic evaluation. 

 

313; Potentially fatal ventricular arrhythmias can occur with concomitant use of cisapride and: 
A. Astemizole 
B. Digoxin 
C. Terbinafine 
D. Erythromycin 
E. Atorvostatin 

 

Co-administration of erythromycin with the antihistamines terfenedine and astemizole or the gastrointestinal promobility agent cisapride increases the risk of torsade de pointes and is contraindicated. 

 

314; Which of the following polypeptides is found in the lamina lucida? 
A. plakoglobin 
B. desmoplakin 
C. keratocalmin 
D. demoyokin 
E. laminin 5

 

Laminin-5 is a basement membrane extracellular matrix protein that mediates attachment substrate for both adhesion and migration in a wide variety of cell types, including epithelial cells, fibroblasts, neurons and leukocytes and is a preferred adhesion substrate for epithelial cells (Koshikawa et al., 2001). The remaining listed items are part of the desmosomal plaque in the epidermis. 

 

 

315; A 36-year-old woman presents with arthralgias and three fingernails exhibiting adherence of the ventral surface of the distal nail plate to the hyponychium. What is the most likely associated disease: 
A. Psoriasis 
B. Lichen Planus 
C. Systemic Sclerosis 
D. Yellow Nail Syndrome 
E. Multicentric Reticulohistiocytosis 

 

The most common cause of ventral pterygium is systemic sclerosis, which often first presents with arthralgias. Lichen planus is most commonly associated with dorsal, rather than ventral, pterygium. Multicentric reticulohistiocytosis is associated with deforming arthritis and “coral bead” nodules around the proximal nail fold, but not with pterygium 

 

316; Which of the following lesions are ”better felt than seen”: 
A. Seborrheic keratosis 
B. SCC in situ 
C. Actinic keratosis 
D. Sebaceous hyperplasia 
E. All of these answers are correct 

 

Typical AKs appear as flat, erythematous, rough papules that are better felt than seen 

 

317; Fomivirsen is a single-stranded antisense oligonucleotide FDA-approved for the treatment of: 
A. HSV infection 
B. HIV infection 
C. EBV infection 
D. CMV infection 
E. HPV infection 

Fomiversen (Vitraene) represents a new class of therapeutic agents known as antisense drugs. It is indicated for the treatment of CMV retinitis in patients with AIDS. The most common adverse effects are ocular inflammation and increases in intraocular pressure. 

 

318; A 80-year-old lady presents to dermatology clinic complaining of itchy white plaques affecting her vulva. There is no history of vaginal discharge or bleeding. A similar plaque is also seen on her inner thigh. What is the likely diagnosis? 
A. Candida 
B. Lichen planus 
C. Lichen sclerosus 
D. Herpes simplex 
E. Seborrhoeic dermatitis 

 

 Lichen sclerosus Lichen sclerosus was previously termed lichen sclerosus et atrophicus. It is an inflammatory condition which usually affects the genitalia and is more common in elderly females. Lichen sclerosus leads to atrophy of the epidermis with white plaques forming 

 

319; Signs and symptoms of lidocaine toxicity include all of the following except: 
A. Circumoral numbness 
B. Ototoxicity 
C. Slurred speech 
D. Nystagmus 
E. Seizure 

 

The first signs of lidocaine toxicity are CNS symtpoms that resemble inebriation with alcohol. These symptmoms include stupor, dysarthria, circumoral numbenss and dizziness. Further increases in toxicity leads to nausea, metallic taste, twitching, and seizures. Ototoxicity is not one of the toxic manifestations seen with lidocaine toxicity. 

 

320; Which of the following is a low sedation metabolite of hydroxyzine? 
A. Fexofenadine 
B. Cetirizine 
C. Loratadine 
D. Cyproheptadine 
E. Ranitidine 

 

Cetirizine is a second-generation H1 antihistamine that is a low sedation metabolite of hydroxyzine 

 

321; The diagnosis of leishmaniasis can be confirmed by which of the following techniques?
A. Culture in Sabourad's agar
B. Warthin-Starry stain
C. Culture in Michel's medium
D. Weil-Felix test
E. Culture in Novy-MacNeal-Nicolle (NNN) medium 

 

Confirmation of infection with Leishmaniasis is performed by culture in Novy-MacNeal-Nicolle (NNN) medium 

 

323; Which of the following would one find cutaneous hyperpigmentation, blue lunulae and Kayser-Fleishcher rings
A. Hemochromatosis
B. Osteogenesis Imperfecta
C. Marfan's Disease
D. Gaucher's Disease
E. Wilson's Disease 

 

In Wilson's disease (Hepatolenticular Degeneration) one will find a vague greenish discoloration of the skin on the face, neck, and gentalia Hyperpigmentation), azure lunulae (sky-blue moons) of the nails, and Kayser-Fleischer rings. This is due to the body retaining excessive amounts of copper 

 

324; A 67-year-old man with a history of Parkinson's disease presents due to the development of an itchy, red rash on his neck, behind his ears and around the nasolabial folds. He had a similar flare up last winter but did not seek medical attention. What is the most likely diagnosis? 
A. Levodopa associated dermatitis 
B. Seborrhoeic dermatitis 
C. Flexural psoriasis 
D. Acne rosacea 
E. Fixed drug reaction to ropinirole 

 

Seborrhoeic dermatitis is more common in patients with Parkinson's disease Seborrhoeic dermatitis in adults Seborrhoeic dermatitis in adults is a chronic dermatitis thought to be caused by an inflammatory reaction related to a proliferation of a normal skin inhabitant, a fungus called Malassezia furfur (formerly known as Pityrosporum ovale). It is common, affecting around 2% of the general population 

 

325; Kamino bodies found in
A Spitz naevus
B Malakoplakia
C Xanthomas
D Ochronosis
E Lepromatous leprosy 

 

Kamino bodies eosinophilic hyaline bodies along dermoepidermal junction found in Spitz naevus. 

 

326; A 58 year old female with lifelong Type 1 diabetes mellitus has end-stage renal failure. What is the most common cutaneous manifestation of end stage renal disease? 
A. Pruritus 
B. Nephrogenic systemic fibrosis 
C. Hyperpigmentation 
D. Acquired perforating dermatosis 
E. Skin pallor 

 

There are may cutaneous manifestations of end-stage renal disease. Chronic anemia can cause skin pallor. Deposition of carotenoids can give the skin a yellowish hue. Photo-distributed hyperpigmentation and ecchymoses are also commonly seen. The most common cutaneous manifestation, however, is pruritus. Other cutaneous findings include metastatic calcification, acquired perforating dermatosis, and nephrogenic systemic fibrosis 

 

327; Which site of the body has the highest penetration ability when topical medications applied 
A. Face 
B. Eyelids 
C. Scrotum 
D. Mucous membrane 
E. Upper arms and legs 

 

D Drug penetration varies depending on body site and thickness of stratum corneum. An important consideration in topical therapy is that abnormal skin may have an altered (increased, decreased, or absent) stratum corneum, thus changing the body site's barrier function. Regional penetration of topical therapy is ranked from highest penetration to less penetration as following: Mucous membrane > scrotum > eyelids > face > chest and back > upper arms and legs > dorsal hands and feet > palms and soles > nails . 

 

328; The most common side effect of azathioprine is: 
A. Bone marrow suppression 
B. Neuropathy 
C. Hepatotoxicity 
D. Nephorotoxicity 
E. Myopathy 

 

The most common side effect of azathioprine (Imuran) is bone marrow suppresion (Pancytopenia). It is a rare adverse event resulting from excessive immunosuppression by azathioprine. To prevent catastrophic bone marrow failure, regular complete blood counts should be checked. 

 

329; Sezary syndrome: 
A. Has characteristic Sezary cells in peripheral blood
B. The five year survival is estimated to be between 10-20%. 
C. Is characterized by the triad of pruritic erythroderma, generalized lymphadenopathy, and presence of Sezary cells in peripheral blood 
D. Is associated with a poor prognosis 
E. All of these answers are correct 

 

Sezary syndrome is characterized by the triad of pruritic erythroderma, generalized lymphadenopathy, and presence of Sezary cells in peripheral blood. Sezary cells are abnormal, large hyperconvoluted lymphocytes. The five year survival is estimated to be between 10-20% 

 

330; Which of the following statements about arthroconidia is correct? 
A. Arthroconidia are formed by budding 
B. Arthroconidia are formed by fragmentation of hyphae 
C. Arthroconidia are thick-walled round cells 
D. Arthroconidia are spores that are produced in a sac 
E. Arthroconidia are yeast forms of dimorphic fungi 

 

B. Arthroconidia are formed by fragmentation of hyphae and may appear as thick or thin walled rectangular cells. An example would be the mould form of Coccidioides immitis. The remaining options are incorrect 

 

331; Immediate pigment darkening: 
A. Is associated with an increase in melanocyte number 
B. Is predominately brought on by UVB 
C. Start 45-60 minutes after exposure 
D. Is predominately brought on by UVA and visible light 
E. Is caused by an increase in tyrosinase activity 

 

Immediate pigment darkening appears almost as soon as irradiation occurs. It is due to photooxidation of preexisting melanin. 

 

332; Which of the following skin conditions is not associated with diabetes mellitus? 
A. Necrobiosis lipoidica 
B. Sweet's syndrome 
C. Granuloma annulare 
D. Vitiligo 
E. Lipoatrophy 

 

Sweet's syndrome is also known as acute febrile neutrophilic dermatosis has a strong association with acute myeloid leukaemia. It is not associated with diabetes mellitus 

 

333; Cronkhite-Canada is associated with gastrointestinal polyposis, nail atrophy, alopecia, generalized pigmentation of skin and melanotic macules on the fingers. Which of the following describes its inheritance pattern?
A. X-linked dominant
B. X-linked recessive
C. Autosomal recessive
D. Autosomal dominant
E. Sporadic 

 

Cronkhite-Canada is associated with gastrointestinal polyposis, nail atrophy, alopecia, generalized pigmentation of skin and melanotic macules on the fingers. It is inherited in a sporadic manner. 

 

334; Which one of the following conditions is least associated with pruritus? 
A. Pemphigus vulgaris 
B. Iron-deficiency anaemia 
C. Polycythaemia 
D. Chronic renal failure 
E. Scabies

 

Pemphigus vulgaris is an autoimmune bullous disease of the skin. It is not commonly associated with pruritus

 

335; An infant with failure to thrive has multiple xanthomas on skin exam and foamy histiocytes on bone marrow biopsy. Your diagnosis is:
A. Niemann-Pick disease
B. Fabry’s disease
C. Tay-Sach’s disease
D. Hunter’s syndrome
E. Gaucher’s disease 

 

disease caused by mutations in sphingomyelin phosphodiesterase-1. Patients with Type A Niemann-Pick disease may have xanthomas, progressive psychomotor deterioration, hepatosplenomegaly, blindness, cherry red spots, and deafness 

 

336; On electron microscopy, which cell demonstrates cytoplasmic projections and secretory granules?
A. macrophage
B. Langerhans cell
C. keratinocyte
D. mast cell
E. melanocyte 

 

Electron microscopy of mast cells demonstrates large long villi at the periphery and round or oval secretory granules. Langerhans cells show a folded nucleus and rarely phagocytized melanosomes on electron microscopy. Their characteristic feature is the presence of Birbeck granules whose disk shape with one or two vesicles at either end represents a tennis raquet. Examination of melanocytes shows an absence of tonofilaments or desmosomes with characteristic melanosomes in various stages of formation. Macrophages often contain phagocytized material within phagosomes. 

 

 

337;   Cefaclor has been associated with increased incidence of what in children? 
A. Mononucleosis-like syndrome 
B. Anaphylaxis 
C. Transaminitis 
D. Serum sickness reaction 
E. Generalized tonic-clonic seizures 

 

The use of cefaclor has been associated with an increased incidence of serum sickness in children 

 

338; The causative agent of Roseola is:
A. A dsRNA virus
B. A ssDNA virus
C. Streptococcus
D. A dsDNA virus 
E. A ssRNA virus 

 

Roseola is caused by Human Herpesvirus 6, a double stranded DNA virus 

 

339; A 50-year old woman develops erythema and swelling around her eyelids and symmetric weakness of her shoulders and hips. What malignancy is overrepresented in patients with this condition compared to the general population? 
A. Ovarian Cancer 
B. Lung Cancer 
C. Leukemia 
D. Uterine Cancer 
E. Breast Cancer 

 

This patient has a heliotrope rash and proximal muscle weakness, consistent with the diagnosis of dermatomyositis. 18-32% of patients with dermatomyositis will develop malignancy. The risk of malignancy is greatest within the first 3 years of diagnosis. Ovarian cancer is overrepresented. 

 

340; Tetracyclines have a risk of photosensitivity. The tetracycline that has the highest risk for photosensitivity is: 
A. Demeclocycline 
B. Doxycycline 
C. Tetracycline 
D. Minocycline 
E. Oxycycline 

 

Tetracyclines bind the bacterial at the 30S ribosomal subunit. There is a risk of photosensitivity with this class of drug. Demeclocycline has the highest risk of photosensitivity 

 

341; Which of the following bones will develop the most dramatic changes in density following oral glucocorticoids? 
A. Femur of 72 year old female 
B. Femur of 72 year old male 
C. Vertebrae of 22 year old male 
D. Vertebrae of 22 year old female 
E. Vertebrae of 72 year old female 

 

Although post menopausal women are at greatest risk for fractures from steroid therapy (they have the weakest bones to begin with), it is young men who have the most dramatic decrease in bone density. Young men have the highest bone density on average and therefore will show the greatest change with steroid therapy. Trabecular bone (ribs, vertebrae) are more affected than cortical bones (long bones) due to higher metabolic rate of the former. Also, most demineralization occurs during the first 6-12 months of therapy. This highlights the fact that even young men who are not traditionaly at risk for fractures, need prophylaxis for osteoporosis while on systemic steroids. 

 

342; Which of the following is NOT a cause of a saddle nose deformity? 
A. Relapsing polychondritis 
B. Hypohidrotic ectodermal dysplasia 
C. Congenital syphilis 
D. Trichorhinophalangeal syndrome 
E. Wegener's granulomatosis 

 

Trichorhinopharyngeal syndrome is the only condition from the mentioned conditions that is not associated with saddle nose deformity.It is associated with sparse hair, a bulbous (pearshaped) nose and cone-shaped epiphyses. 

 

343; Mutations of the p53 gene has been associated with the development of: 
A. Melanoma 
B. Kaposi's sarcoma 
C. Actinic keratosis 
D. Molluscum contagiosum 
E. All of these answers are correct 

 

UVB radiation triggers the formation of thymidine dimers both in DNA and RNA, resulting in mutated keratinocytes. The mutations occur on the tumor suppressor gene p53 within the keratinocytes resulting in impairment of the mechanism of apoptosis. Therefore, clonal expansion of mutated keratinocytes may occur leading to the formation of AKs. 

 

344; The dominant organism present on oily regions of the skin is: 
A. Gram positive coagulase negative staphylococcus
B. Staphylococcus aureus 
C. Propionibacterium acnes 
D. Corynebacterium spp. 
E. Propionibacterium avidum 

 

The most common organism on sebaceous regions of human skin is Propionibacterium acnes. This is an aerotolerant, anaerobic gram positive club-shaped diptheroid that lives in the depths of the sebaceous follicle. It has a potent lipase which cleaves sebaceous triglycerides into glycerol and free fatty acids. The free fatty acids lower the pH and are thought to retard the growth of pathogens such as S. aureus. Other residents of oily regions are: Malassezia furfur, S. epidermidis, Micrococcus luteus and other catalase positive aerobic cocci. S. aureus is a pathogen, and is not considered normal flora of any region of the skin. The other options are found on dry or moist regions of the skin. 

 

346; All of the following are true regarding polymorphous light eruption except: 
A. Pruritic 
B. Abnormal metabolism of arachidonic acid 
C. Hardening occurs with subsequent episodes 
D. Lesions heal without scarring 
E. Anti-Ro antibody positive 

 

Polymorphous light eruption is the most common photodermatosis that is characterized clinically by the abnormal occurrence of pruritic, erythematous, edematous papules following exposure to UV radiation. Lesions heal without scarring. It tends to affect women 2-3x more than men. Positive Anti-ro antibodies should raise the suspicious for subacute cutaneous lupus erythematosus (tends also to be less pruritic). 

 

347; What nerve innervates the skin between the 1st and 2nd toes? 
A. saphenous nerve 
B. posterior tibial nerve 
C. sural nerve 
D. superficial peroneal nerve 
E. deep peroneal nerve 

 

deep peroneal nerve innervates the skin between the 1st and 2nd toes. The saphenous innervates the instep and medial ankle. The posterior tibial innervates the heel and middle sole of the foot. The sural nerve innervates the 5th toe and lateral side of the sole. The superficial peroneal nerve innervates the skin of the toes other than the outside of the 5th toe and in between the 1st and 2nd toes. 

 

348;  patient with diffuse severe sudden hair loss developing 3 months after hospitalization for septicemia likely has which of the following conditions?
A. Anagen effluvium 
B. Catagen effluvium 
C. Telogen effluvium 
D. Loose anagen syndrome 
E. Uncombable hair syndrome

 

Telogen effluvium typically ensues stressful events after a 2-3 month period 

 

349; The most common complication for sarcoidosis is: 
A. Anterior uveitis 
B. Glaucoma 
C. Pars planitis 
D. Diplopia 
E. Paresis if ocular muscles 

 

The most common eye complication of sarcoidosis is anterior uveitis with mutton fat keratotic precipitates. All the others can be a complication of sarcoidosis also. 

 

350; This type of psoriasis is associated with more severe body psoriasis. The location of this type is: 
A. Centrofacial psoriasis 
B. Palmoplantar psoriasis 
C. Scalp psoriasis 
D. Nail psoriasis 
E. Psoriatic arthritis 

 

Centrofacial psoriasis may be associated with more severe body psoriasis. Palmoplantar psoriasis can be difficult to treat. Patients that have psoriatic arthritis can be candidates for biologic therapy. 

 

351; The most common site for intra oral melanoma is? 
A. buccal 
B. soft palate 
C. hard palate 
D. gingiva 
E. uvula 

 

C Multiple case series have shown that the hard palate (and specifically the anterior hard palate/alveolar arch) is the highest risk location for intra-oral melanoma. Prognosis is generally worse for oral melanoma, which is usually due to delay in diagnosis and presentation with more invasive disease 

 

352; Mast cells are derived from bone marrow __+ cells? 
A. CD3 
B. CD6 
C. CD20 
D. CD34 
E. CD68  

 

CD34+ cells in the bone marrow are the precursors of mast cells. CD3 and CD20 are T and B cell markers respectively. CD6 is found on mononuclear phagocytic cells in the dermis, CD68 is a macrophage marker 

 

353; Activation of the procerus muscle causes 
A. Periocular wrinkles 
B. Wrinkles at the nasal root 
C. Wrinkles on the forehead 
D. Perioral wrinkles 
E. Accentuation of the melolabial folds 

 

The procerus muscle causes wrinkling at the nasal root and is often targeted with Botox therapy for improved cosmesis. 

 

354; Which of the following has been implicated in the promotion of skin aging? 
A. alpha-tocopherol 
B. homocysteine 
C. proanthocyanidin 
D. beta carotene 
E. alpha-lipoic acid 

 

Homocysteine is a sulfhydryl-containing amino acid produced during the conversion of methionine to cysteine. Homocysteine can promote defects in fibrillin-1 and stimulate matrix metalloproteinase production, leading to collagen and elastic fiber degradation. Homocystinuria, caused by a deficiency in cystathionine beta-synthase, presents with osteoporosis, thin skin, and striae. 

 

355; Which of the following statements about elastic fibers is true? 
A. Elastic fibers form 35% of the dry weight of the skin 
B. Elastic fibers are 90% elastin wrapped in fibrillin 
C. Collagen 1 is mutated in Marfan syndrome 
D. Oxytalan fibers run parallel within the superficial papillary dermis 
E. Elaunin fibers run perpendicular in thin bands within the reticular dermis 

 

B Elastic fibers are responsible for much of the elasticity of the dermis. They are essentially 90% elastin wrapped in fibrillin. They form 4% of the dry weight of the skin. Fibrillin 1 is mutated in Marfan syndrome. Oxytalan fibers run PERPENDICULAR from the DEJ within the superficial papillary dermis. Elaunin fibers run parallell in thin bands within the reticular dermis. 

 

356; What family of medications is associated with xerosis? 
A. Sulfonylureas 
B. Beta blockers 
C. Calcium channel blockers 
D. Loop diuretics 
E. Cholesterol lowering agents 

 

Medications that alter the lipid composition of the epidermis and stratum corneum may impair the normal barrier function of the skin. Cholesterol lowering medications like HMG-CoA reductase inhibitors and niacin may cause xerosis through this mechanism. 

 

357; Nevus of Ota is also known as nevus fuscoceruleus ophthalmomaxillaris. It is usually present at birth in the two-thirds of patients who have ocular involvement. Other lesions may not appear until the teen years. The conjunctiva and skin about the eye supplied by the first and second branch of the trigeminal nerve, as well as the sclera, ocular muscles, retrobulbar fat, periosteum, and buccal mucosa, may be involved. On the skin, brown, slate gray, or blue–black macules grow slowly larger and deeper in color . It persists throughout life. Eighty percent occur in females; 5% are bilateral. Glaucoma or ipsilateral sensorineural hypoacusia may also occasionally complicate nevus of Ota. Malignant melanoma rarely occurs in nevus of Ota. Q-switched lasers have been used successfully to treat nevus of Ota. Nd:YAG laser at 1064 nm is suitable for use in a wide range of skin types. 

 

358; All of the following retinoids are excreted in the urine EXCEPT: 
A. Bexarotene 
B. Etretinate 
C. Isotretinoin 
D. Tretinoin 
E. Acitretin 

 

Bexarotene is excreted via hepatobiliary excretion. The others are excreted in bile and urine 

 

360; The standard of care of patients with acute Kawasaki’s disease is:
A. Supportive care
B. Aspirin and IVIG
C. Penicillin
D. Acetaminophen and IVIG
E. Prednisone 

 

Kawasaki disease, also called mucocutaneous lymph node syndrome, is an acute febrile disorder based on the clinical criteria of changes in peripheral extremities, polymorphous exanthema, conjunctival injection without exudates, changes in the lips or oral cavity, acute cervical lymphadenopathy. Fever must be present, lasting more than 5 days. Treatment is aimed to prevent coronary aneurysms and myocardial infarction. Treatment for acute Kawasaki disease is intravenous immunoglobulin 2 g/kg over 10-12 hours and aspirin therapy. 

 

361; A 26-year-old woman presents with a rash. She describes the rash as occurring in crops with lesions tending to self resolve within a few weeks. On examination she has multiple erythematous to purple crusty papules with some small ulcers, vesicles and pustules. In some areas where lesions have resolved varioliform scarring has been left behind. A biopsy is taken that shows an interface dermatitis with necrotic keratinocytes, T-cell clonality studies show a predominantly CD8+ monoclonal infiltrate. What is the most likely diagnosis:
A pityriasis lichenoides et varioliformis acuta (PLEVA) 
B pityriasis lichenoides chronic (PLC) 
C mycosis fungoides 
D guttate psoriasis 
E small plaque parapsoriasis. 

 

Pityriasis lichenoides et varioliformis acuta (PLEVA) and pityriasis lichenoides chronic (PLC) are considered to be two ends of the same disease spectrum. Both are characterised by recurrent crops of selfresolving erythematous papules. The lesions of PLEVA are more acute and tend to be crusty, vesicular and pustular whereas PLC tends to be more chronic and scaly. In reality few patients are at one end of the spectrum and most show a mixture of features. T-cell clonality is seen in both disorders and they may be considered as T-cell lymphoproliferative disorders. Treatment consists of topical steroids, coal tar products, antibiotics and light therapy. The recurrent crops of PLEVA/PLC are quite characteristic and exclude other conditions such as mycosis fungoides and guttate psoriasis. Parapsoriasis is a related T-cell lymphoproliferative disorder that is considered by some to be a precursor to mycosis fungoides. It presents with chronic, asymptomatic scaly patches. 

 

362; Which of the following is the vector of lymphatic filariasis caused by Wuchereria bancrofti?
A. Black fly
B. Deerfly
C. Mosquito 
D. Botfly
E. Tsteste fly 

 

Lymphatic filariasis is spread by mosquitoes belonging to genera Aedes, Anopheles, Culex, or Mansonia. 

 

363;   Laugier–Hunziker syndrome (LHS) is an acquired pigmentary condition affecting lips, oral mucosa and acral area, frequently associated with longitudinal melanonychia. There is neither malignant predisposition nor underlying systemic abnormality associated with LHS. 

 

364; Which of the following is true regarding neonatal lupus erythematosus? 
A. Most cases involve boys 
B. Lesions generally resolve spontaneously by 6 months, healing with scarring 
C. Photosensitivity is generally not a feature 
D. 75% of mothers have symptomatic systemic lupus erythematous at the time of delivery 
E. Congenital heart block may be the only manifestation of the disease 

 

Neonatal LE presents with annular scaling erythematous macules and plaques on the head and extremities within the first few months of life in babies born to mothers with LE, rheumatic diseases, or other connective tissue disorders. 50% of mothers are asymptomatic at delivery. Lesions resolve spontaneously by 6 months, healing without scarring. Photosensitivity may be prominent. 75% of cases involve girls. 50% have congenital heart block, which is permanent, and may be the only manifestation of the disease. Thrombocytopenia and hepatic disease are as frequent as cardiac disease. 

 

 

364;  Irritation of the hand produced by capsaicin can be relieved by application of 
A. Water 
B. Acetic acid 5% 
C. Sodium chloride 
D. Alkali 
E. Talcum powder 

 

Burning secondary to capsaicin can be relieved by vinegar (acetic acid 5%) as the capsaicin is soluble in vinegar (but not water). 

 

365;  Periungual Squamous cell carcinoma has been linked to which HPV type(s)?
A. 6, 11 
B. 2, 4 
C. 16 
D. 13 
E. 8 

 

Infections associated with Human Papilloma Virus can produce growths on the epithelial or mucosal surfaces. There are over 100 strains of these viruses, and some of these strains can predispose to intraepithelial carcinomas, particularly when involving the anal or genital mucosa. In general, HPV strains 16 and 18 are classified as his risk virus types and can be associated with cervical cancer , oral cancer, anal cancer and periungual cancers. HPV 6 and 11 are associated with condyloma acuminata; HPV types 2, 4 are associated with common warts; HPV type 13 has been associated with Heck's disease and HPV type 8 has been associated with epidermal dysplasia verruciformis 

 

 

366; A 62-year-old with a history of acne rosacea presents for advice regarding treatment. Which one of the followinginterventions has the least role in management? 
A. Camouflage creams 
B. Topical metronidazole 
C. Low-dose topical corticosteroids 
D. Laser therapy 
E. Use of high-factor sun block 

 

Management 

-topical metronidazole may be used for mild symptoms (i.e. Limited number of papules and pustules, no plaques) 
-more severe disease is treated with systemic antibiotics e.g. Oxytetracycline 
-recommend daily application of a high-factor sunscreen 
-camouflage creams may help conceal redness 
-laser therapy may be appropriate for patients with prominent telangiectasia 

 

367;  The mechanism of action of mycophenolate mofetil most closely resembles that of what other drug? 
A. Cyclophosphamide 
B. Doxorubicin 
C. Azathioprine 
D. Hydroxyurea 
E. Chlorambucil 

 

Both azaithioprine and mycophenolate mofetil directly interfere with purine synthesis. 

 

368; Which of the following dermatoses occurs at the latest stage of pregnancy? 
A. Darkening of nevi 
B. Linea nigra 
C. Melasma 
D. Areolar hyperpigmentation 
E. Psoriasis exacerbation 

 

Linear nigra is one the latest skin changes in pregnancy. It is a hyperpigmented, linear streak which extends from the pubic symphysis to the xiphoid process. It usually appears from the 20th week. 

 

369;  Twenty-nail dystrophy, nail plate splitting and pterygium formation are nail changes seen in: 
A. Lichen Planus 
B. Darier's disease 
C. Psoriasis 
D. Scleroderma 
E. Dermatomyositis 

 

Twenty-nail dystrophy, nail plate splitting and pterygium formation are nail changes associated with Lichen Planus. Darier's disease is associated with longitudinal red and white streaks of the nail plate and V-shaped knicking distally. Psoriasis is associated with many nail abnormalities including pitting, onycholysis and oil-spots. Scleroderma may cause nail fold capillary dilation and destruction while patients with dermatomyositis may exhibit nail fold telangiectasias and frayed cuticles 

 

370;  A 4 year-old boy is diagnosed with Staphylococcal Scalded-Skin Syndrome. Vesiculation in this disease is secondary to exfoliative toxins binding what target protein? 
A. Plectin 
B. Bullous pemphigoid antigen 2 
C. Bullous Pemphigoid antigen 1 
D. Desmoglein 3 
E. Desmoglein 1 

 

Staphylococcus Scalded-Skin Syndrome (SSSS) is caused by Staphylococcus exfoliative toxins A and B binding desmoglein 1. This results in the intraepidermal split at the granular layer seen in this illness. Targeting of desmoglein 3 would result in a suprabasilar split. Targeting of plectin, BPAG1, or BPAG2 would result in a subepidermal split. This is not seen in SSSS 

 

368; Tick-borne Relapsing Fever is caused by: 
A. B. burgdorferi 
B. B. recurrentis 
C. B. duttonii 
D. B. henselae 
E. B. bacilliformis 

 

Relapsing Fever can be either Louse-borne or Tick-borne. Tick-borne Relapsing Fever is caused by B. duttonii or B. hermsi, while Louse-borne Relapsing Fever is caused by B. recurrentis.  

 

369; Keratoacanthomas have been linked etiologically to:
A. Ultraviolet exposure 
B. Human papilloma virus 
C. Chemical carcinogens such as tar and pitch 
D. Smoking 
E. All of these answers are correct 

 

The origin of KAs has not been established. Ultraviolet exposure, exposure to chemical carcinogens such as tar and pitch, as well as smoking, and a viral etiology, specifically the human papilloma virus, have all been proposed as etiologic factors 

 

370; Which of the following is NOT true about UVB radiation? 
A. Responsible for sunburn 
B. More intense in the summer than winter months 
C. Peaks at noon 
D. Decreased with high wind velocity 
E. Virtually all blocked by car window glass 

 

D. The UVB band extends from 290 to 320 nm. The UVB spectrum is recognized as the primary cause of sunburn, skin cancer, and other harmful effects on human skin. The UVA band extends from 320 to 400 nm. This spectrum is further subdivided into UVA-2 (320 to 340 nm) and UVA1 (340 to 400 nm). The UVA spectrum is recognized as a cause of immediate and delayed tanning reaction of skin, and several other effects including photoaging, skin photosensitization, and immunosuppression. UVC radiation comprises wavelengths shorter than 290 nm (from 200 to 290 nm). Notably, window glass filters out ultraviolet wavelengths shorter than 320 nm, so both UVB and UVC are effectively filtered by car window glass. UVB radiation is more intense during summer months compared to winter months and peaks during midday hours. It has been postulated that physical factors such as high temperature, high humidity, and wind can all increase susceptibility to UV-induced carcinogenesis.  

 

371;  Acute hemorrhagic edema of childhood affects children and infants < 2 years of age. It presents with painful, edematous petechiae and ecchymoses on the head and distal extremities. Facial edema may be the initial sign. Triggering factors include infection, drugs, and immunization. It lacks the systemic features of HSP, and resolves in 1-3 weeks without sequelae.  

 

372; Which of the following chemotherapeutic agents has been linked to acneiform eruptions? 
A. Bleomycin 
B. Cytarabine 
C. Cisplatin 
D. Doxorubicin 
E. Cetuximab 

 

Cetuximab is a chimeric anti-epidermal growth factor receptor antibody that is FDA approved to treat advanced colorectal cancer. Acneiform eruptions have been reported to occur in up to 1/3 of patients   

 

373;  Which of the following vehicles is correctly defined 
A. Ointment-water in oil emulsion 
B. Gel-oil in water emulsion 
C. Cream-Semisolid emulsion in alcohol base 
D. Lotion/Solution-Cream in water 
E. Foam - powder in cream

 

Various vehicles are used for different clinical situations. Ointments are water in oil emulsions. Creams are oil in water emulsion. Gels are semisolid emulsion in alcohol base . Lotions/Solutions are powder in water (some oil). Foams are liquid and/or solid materials in a gaseous medium. e. All are correct (false  

 

374;  Which is the following is true regarding cellulite? 
A. It is caused by lipoatrophy of fat. 
B. Adipocytes in the gluteofemoral region are more responsive to lipolysis. 
C. Norepinephrine is the only hormone that affects lipolysis. 
D. It occurs in up to 50% of postpubertal females. 
E. Caucasian women tend to get cellulite more than Asian women. 

 

According to the JAAD CME article in March 2010, cellulite is caused by the herniation of subcutaneous fat, not lipoatrophy, within fibrous connective tissue. It is prevalent in almost all postpubertal females. It is most notable in the pelvic region, lower limbs, and abdomen. The adipocytes in the gluteofemoral region are larger and are influenced by female sex hormones. They are metabolically more stable and resistant to lipolysis. The hormones that acutely affect lipolysis in adipocytes are catecholamines (epinephrine and norepinephrine) and insulin.   

 

375; A specific marker of Merkel cells is: 
A. Cytokeratin 10 
B. Cytokeratin 15 
C. Cytokeratin 20 
D. Loricrin 
E. Envoplakin 

 

Cytokeratin 20 is a specific marker for the Merkel cell. Merkel cells are mechanoreceptors located at body sites requiring high tactile sensitivity. Keratinocyte deformation results in a secretion of chemokines by Merkel cells, which make synaptic connection with neurons. 

 

376; Which of the following is associated with Reiter’s syndrome: 
A. Ulcerative colitis 
B. Multiple sclerosis 
C. HLA-Cw6 
D. Asymmetric arthritis
E. Perioral dermatitis 

 

D. Reiter’s syndrome is a chronic inflammatory disease similar to psoriatic arthritis. Commonly, patients present with either a peripheral, asymmetric arthritis, or a urethritis, or conjunctivitis. Not all of these findings need be present. Patients are usually men with the HLA-B27 genotype. Other clinical findings include fever, weight loss, keratitis, iritis, and cardiac disease. Skin lesions are often found on the palms (keratoderma blenorrhagicum _ hyperkeratotic papules and plaques) or the penis (balanitis circinata). Reiter’s follows an infection of either the genitourinary tract or the gastrointestinal tract   

 

377; Pediculus humanus var. corporis (human body louse) is the vector in which of the following diseases:
A. Q fever (C. burnetii)
B. Endemic typhus (R. typhus)
C. Rickettsialpox (R. akari)
D. Epidemic typhus (R. prowazekii)
E. Rocky Mountain Spotted Fever (R. rickettsii) 

 

D The human body louse is the implicated Trench fever, epidemic typhus, and relapsing fever. Trench fever usually affects alcoholic men and is manifested by fevers. Treatment is with ceftriaxone, erythromycin, or doxycycline. Epidemic typhus is manifested by fevers, chills, malaise, and a pink macular eruption beginning in the axilla and trunk. Treatment is with tetracycline or chloramphenicol. Relapsing fever is manifested by paroxysmal fevers, headache, lymphocytoma, and erythematous/petechial macules on trunk/extremities. Treatment is with doxycycline   

 

378; The treatment of choice of cat-scratch disease is:
A. Itraconazole
B. Mebendazole
C. Ceftriaxone
D. Trimethoprim-sulfamethoxazole
E. Erythromycin 

 

Cat-scratch disease is caused by Bartonella henselae, which is spread among cats by fleas. The hallmark of the disease is unilateral and regional lymphadenopathy. The treatment of choice for cat-scratch disease is erythromycin 

 

379; What is the average duration of the telogen cycle in terminal scalp hair?
A. 9 months
B. 6 months
C. 2-3 weeks
D. 2-6 years
E. 3 month 

 

The average duration of the telogen phase of the hair cycle is 3 months; this feature explains why telogen effluvium is typically observed 3 months following a traumatic event or serious illness. The average duration of the anagen phase of the hair cycle is 2-6 years, whereas that of the catagen cycle is 2-3 weeks 

 

380; Which of the following locations has eccrine glands?
A. under nails
B. labia majora
C. penis
D. ear canal
E. labia minora 

 

Eccrine glands are enervated by cholinergic sympathetic nerves, mediated by acetycholine. They are absent on modified skin that lack appendages such as the vermillion border, nail bed, labia minora, glans penis and ear canal. They are present of the labia majora.   

 

381: A patient with a innumerable disseminated keratoacanthomas, including lesions on the larynx and oral mucosa: 
A. Is unlikely to have palmoplantar involvement 
B. Likely has an underlying immune deficiency 
C. Is at high risk for myelodysplasia 
D. Likely inherited their condition in an autosomal dominant manner 
E. Likely developed them during adulthood 

 

 

This patient has the Grzybowski type of keratoacanthomas. Typically diagnosed in adulthood, these patients have the sudden appearance of hundreds of small lesions in a disseminated fashion. The lesions can be found anywhere on the body including palms, soles, larynx, and oral mucosa.  

 

 

382;  Which of the following skin findings is most closely linked to hepatocellular carcinoma as a paraneoplastic syndrome? 
A. Pityriasis lichenoides 
B. Pityriasis alba 
C. Pityriasis amiantacea 
D. Pityriasis rotunda 
E. Pityriasis rosea 

 

Pityriasis rotunda is an dermatosis that features characteristic discrete, circular, scaly, brown patches on the trunk and extremities. Pityriasis rotunda may be associated with systemic diseases in certain racially predisposed groups (blacks), and has been linked to hepatocellular carcinoma  

 

383;  Which of the following organisms would you expect to recover from a fluorescent tinea capitis? 
A. T. tonsurans 
B. M. gypseum 
C. M. canis and M. gypseum 
D. T. violaceum 
E. T. rubrum 

 

C M. gypseum may occasionally produce a dull fluorescence. M. canis is fluorescent.    

 

384; The characteristics of tuberculoid leprosy or TT is:
A. Lepromin test + 
B. IL4, IL-10, 
C. Multibacillary 
D. Lepromin test - 
E. No loss of sensation or sweating 

 

Tuberculoid leprosy has a TH1 cytokine profile of IFN gamma, IL-2, IL-12. It is paucibacillary, Lepromin test +. There is less than 3 lesions and anesthetic and anhidrotic lesions  

 

385;  The medication most acceptable for usage in patients with renal failure is: 
A. Tetracycline 
B. Minocycline 
C. Demeclocycline 
D. Oxytetracycline 
E. Doxycycline 

Renal failure may prolong the half-life of most tetracyclines except doxycycline. Doxycycline is excreted via the GI tract, unlike the other tetracyclines  

 

386;  Herpes gestationis is most commonly associated with which HLA? 
A. HLA-DR3 
B. HLA-B27 
C. HLA-B51 
D. HLA-DR9 
E. HLA-DQ8 

 

HLA-DR3 is the most commonly found HLA association in herpes gestationis. HLA-DR4 is also found in addition to HLA-DR3 in about 50% of patients. There is nearly 100% incidence of antiHLA antibodies patients affected by herpes gestationis.  

 

387;  This medication is a source of anagen effluvium and can cause abrupt shedding of hair: 
A. Colchincine 
B. Hydrochlorothiazide 
C. Beta blockers 
D. Calcium channel blockers 
E. Procainamide  

 

Colchichine, mercury intoxication, thallium poisoning and severe protein deficiency can cause anagen effluvium. Hairs are usually broken and not shed.  

 

388; What is the most common presentation of mucous membrane involvement of EBV infection? 
A. Buccal ulcers with superficial slough 
B. Erosions of the gingivae 
C. Plication of the tongue 
D. Palatal petechiae 
E. Oral hairy leukoplakia 

 

D. The most common presentation of intra-oral involvement of EBV infection is pinpoint petechiae of the junction of the hard and soft palate. The associated eponym is Forchheimer spots. Forchheimer spots are not pathognomonic for EBV infection, as they can be seen in measles, rubella, and scarlet fever. While oral hairy leukoplakia is associated with EBV infection, it is not the most common mucous membrane sign  

 

389; All of the following are true regarding Cockayne Syndrome EXCEPT: 
A. Inheritance is autosomal recessive 
B. It is caused by an inability to repair cyclobutane dimers 
C. It is associated with basal ganglia calcifications 
D. It is associated with retinal pigment degeneration with a "salt and pepper" appearance 
E. None of these answers are incorrect (all are correct) 

 

Cockayne Syndrome is an autosomal recessive disorder caused by a mutation of an unknown gene that results in the inability to repair cyclobutane dimers induced by UV exposure. Cutaneous findings include photosensitivity, "bird-headed" facies, and "Mickey Mouse" ears. Statements A-D are true regarding the syndrome  

 

390; The risk of metastasis from SCC increases with: 
A. Tumor size 
B. Depth of invasion 
C. Degree of differentiation 
D. Immunosupression 
E. All of these answers are correct 

 

Tumor size (greater than 2 cms), location (lips, ears, eyelids), depth of invasion, degree of differentiation, perineural invasion, immunosupression, as well as recurrent tumors and those that arise in areas of chronic inflammation are all factors that increase the risk of metastasis.  

 

391; Which tattoo pigment has most commonly been associated with phototoxic reactions? 
A. Titanium dioxide 
B. Mercuric sulfide 
C. Carbon 
D. Iron oxide 
E. Cadmium sulfide 

 

Cadmium sulfide produces a yellow tattoo. Phototoxic reactions are most commonly caused by cadmium sulfide tattoo pigment. Red tattoos have also been associated with phototoxic tattoo reactions. However, this is thought thought to be due to the addition of cadmium sulfide which enhances and brightens the red color  

 

392; Imiquimod induces which of the following cytokines? 
A. Interleukin-10 
B. Interferon-alpha 
C. Interleukin-2 
D. Interleukin-4 
E. Interleukin-5 

 

Imiquimod is a topical immunomodulator that increases a variety of cytokines including interferon-alpha  

 

394;  Which of the following medication is a cause of hirsutism without virilization? 
A. Dapsone 
B. Diazoxide 
C. Diazepam 
D. Dinitrochlorobenzene 
E. Dantrolene  

B Phenytoin, diazoxide, cyclosporine and hexachlorobenzene all can cause increased hair growth in patients. In addition, oral (and topical) minoxidil can cause hirsutism. The other listed medications do not cause hirsutism  

 

395;  Meibomian glands are: 
A. Eccrine glands localized to the vermillion border of the lips 
B. Sebaceous glands found on the areola of the breast 
C. Sebaceous glands found on the eyelids 
D. Apocrine glands found in the anogenital regions 
E. Apocrine glands found on the eylelids 

 

C Sebaceous glands enlarge at puberty in response to increased levels of androgens. They are holocrine glands. Meibomian glands are modified sebaceous glands found in the eyelids. Free sebaceous glands not associated with hairs are found in the nipple and areola and are called Montgomery′s tubercles. Fordyce′s condition involves free sebaceous glands on the vermillion border of the lips and on the buccal mucosa. Sebaceous glands are found everywhere on the skin except the palms and soles.  

 

396;  The causative organism that is responsible for pitted keratolysis is: 
A. Micrococcus sedentarius 
B. Streptobacillus moniliformis 
C. Klebsiella pneumonia 
D. Neisseria meningitides 
E. Burkholderia mallei 

 

Pitted keratolysis is caused by Micrococcus sedentarius. The clinical features are shallow 1-3mm pits on plantar surface of feet. The treatment is topical erythromycin, clindamycin, or benzoyl peroxide.  

 

397;  Characteristic eosinophilic abscesses are seen in: 
A. Bullous drug 
B. Pemphigus vegetans 
C. Herpes gestationis 
D. Incontinentia pigmenti 
E. Paraneoplastic pemphigus 

 

Eosinophilic abscesses with minimal to no spongiosis in a hyperplastic epidermis are characteristic of pemphigus vegetans.  

 

398; A patient taking azathioprine for bullous pemphigoid develops a hypersensitivity syndrome characterized by fever and shock. Approximately how long ago did the patient start this medication?
A. 14 days 
B. 24 hours 
C. 3-4 days 
D. 6 weeks 
E. 1 week 

 

A Azathioprine is a purine analog which is used to treat multiple inflammatory disorders, including autoimmune bullous disease, vasculitis, and severe dermatitis. A hypersensitivity syndrome with fever and shock can occur at around 14 days.  

 

399; This antiviral medicine doesn't require phosphorylation and may be used for acyclovir recalcitrant disease: 
A. Foscarnet 
B. Gancyclovir 
C. Famciclovir 
D. Valacyclovir 
E. Penciclovir 

 

Foscarnet is a non-competitive inhibitor of viral DNA polymerase. It does not require phosphorylation and can be used for acyclovir recalcitrant disease.   

 

400; A patient with coarse facies, broad nasal bridge, and extensive eczema might be expected to have which abnormal laboratory value? 
A. Hypertriglyceridemia 
B. High Copper levels 
C. Low Zinc levels 
D. High IgE levels 
E. Anemia  

 

Patients” with Hyper IgE Syndrome (Job syndrome) have impaired regulation of the IgE function and increased susceptibility to infections. In addition to recurrent cutaneous infections including cold abscesses, patients have widespread eczematous dermatitis, recurrent sinopulmonary infections and typically have coarse facies with broad nasal bridge and a prominent nose  

 

401; Which of the following is a characteristic of papilloma viruses? 
A. enveloped viruses 
B. single stranded RNA viruses 
C. replicate independent of the host cell 
D. controlled by host antibody regulated response 
E. can cause a condition that manifests with stridor and hoarseness in children 

 

E Papilloma viruses are non-enveloped,double stranded DNA viruses. Cell mediated responses are primarily responsible for controlling papillomavirus infections. Unlike viruses such as HSV, HPV does not have enzymes required for replication of viral DNA, and is entirely dependent on the host cellular machinery. Recurrent respiratory papillomatosis can be caused by HPV-6 and 11, with verrucous lesions of the airways. It can occur as a juvenile or adult-onset form and present with hoarseness in children. 

 

402;  The most important mutated gene associated with a predisposition to develop malignant melanoma is: 
A. PTCH 
B. CDKN2A 
C. PTCH2 
D. None of these answers are correct 
E. All of these answers are correct 

 

The CDKN2A gene located on chromosome 9p21 is the most important mutated gene associated with MM of the listed choices. BRAF is another important mutation.  

 

403; The patient is a 45 year old male complaining of red, chapped lower lip. Which of the following lasers is the most appropriate to treat this condition? 
A. Pulsed Dye Laser 
B. Nd:YAG laser 
C. CO2 laser 
D. Diode laser 
E. Laser treatment is not an option 

 

The patient has actinic cheilitis. Notice the red, scaly lower lip, with erosions and fissures. The CO2 laser is currently a common treatment alternative for this condition  

 

404;  Which of the following is the most common initial site of metastasis from a primary BCC? 
A. Lungs 
B. Regional lymph nodes 
C. Bone 
D. Liver 
E. Pleura 

 

The metastatic potential of BCC is very low with rates ranging from 0.0028 to 0.1%. The head and neck region is the most frequent location of the primary tumor with regional lymph nodes being the most common site of metastasis. The lungs, bone, liver, and pleural are also potential sites of metastasis  

 

405; The immunoglobulin most commonly found in mucous secretions is: 
A. IgA 
B. IgD 
C. IgE 
D. IgG 
E. IgM 

 

IgA is found in mucous membrane secretions and is able to agglutinate antigens and activate the alternate but not the classic complement pathway. IgG is the antibody that can cross the placenta and the most common antibody found in circulation. IgD is not found in circulation other than in hyper-IgD syndrome, an autosomal recessive disorder caused by mutations in the mevalonate kinase gene. A significant elevation of serum IgD is seen in 95% of these patients. IgE is an anaphylactic antibody that is involved in nearly all immediate allergic and anaphylactic type reactions. IgM is the antibody produced in the early stages of antibody responses. It is a pentamer which can agglutinate antigen and active the classic complement pathway  

 

406;  The muscle that is affected in a patient with a fibromatosis colli is: 
A. Sternocleidomastoid muscle 
B. Masseter muscle 
C. Frontalis muscle 
D. Bicepts 
E. Tricepts 

 

Fibromatosis colli affects the lower sternocleidomastoid muscle that my arise from birth trauma. These occur in the pediatric patients and can spontaneous remit 

 

407; The most common age group for papular-purpuric gloves and socks syndrome is: 
A. Newborns 
B. Toddlers 
C. 6-10 year olds 
D. Adolescents 
E. Elderly 

 

This unique presentation of parvovirus infection typically occurs in adolescents and young adults.  

 

408;  Epidermolysis bullosa simplex is caused by mutations in: 
A. Keratins 1&10 
B. Keratins 1&9 
C. Keratins 5&14 
D. Keratins 6a &16 
E. Keratins K2e&10 

 

EB Simplex is caused by mutations in keratins 5 &14. Mutations in Keratins 1&10 are associated with epidermolytic hyperkeratosis and Unna-thost. Mutations in keratins 1&9 are associated with epidermolytic palmoplantar keratoderma. Mutations in Keratins 6a and 16 are associated with Pachyonychia congenital. Mutations in K2e and 10 are associated with Icthyosis bullosa of Siemens.  

 

408; A patient has multiple erythematous/violaceous psoriasiform dermatitis affecting the ears, nose, hands and feet. The patient is diagnosed with Bazex syndrome. The most appropriate course of action is to: 
A. Do an age appropriate cancer screening. 
B. Use topical corticosteroids 
C. To do a CT scan of the abdomen 
D. To order lipase and amylase levels 
E. To do a CT of the chest 

 

A patient with Bazex syndrome also known as Acrokeratosis Paraneoplastica should have an age appropriate cancer screening. Patients can also have nail dystrophy , acquired keratoderma, and the skin findings usually precede the underlying malignanc

 

Comments

Post a Comment

Popular posts from this blog

My personal MCQ Notes